Win up to 100% Scholarship

Register Now

Indian Economy

 

2022

 

Question 1

With reference to the Indian economy, consider the following statements:

1. If the inflation is too high, the Reserve Bank of India (RBI) is likely to buy government securities.

2. If the rupee is rapidly depreciating, RBI is likely to sell dollars in the market.

3. If interest rates in the USA or European Union were to fall, that is likely to induce RBI to buy dollars.

Which of the statements given above are correct?

(a) 1 and 2 only

(b) 2 and 3 only

(c) 1 and 3 only

(d) 1, 2 and 3

Ans: b

Sub-Theme: Qualitative Tools to control

Statement 1 is incorrect: If the inflation is too high, Reserve Bank of India (RBI) is likely to reduce the money supply in the economy to control inflation. Thus, RBI sells the government securities so as to suck the excess money supply from the economy and to control the inflation.

Statement 2 is correct: RBI intervenes in the currency market to support the rupee because if it becomes a weak domestic unit then it can increase a country’s import bill. It can intervene directly in the currency market by buying and selling dollars. If RBI wishes to prop up rupee value, then it can sell dollars and when it needs to bring down rupee value, it can buy dollars.

Statement 3 is correct: When the US raises its domestic interest rates, this tends to make India less attractive for the currency trade. As a result, it pulls some money from the Indian market out and flows back to the US, thereby decreasing the value of India’s currency against the US dollar. Therefore, if interest rates in the USA or EU were to fall, then it is likely to induce RBI to buy dollars.

 

Question 2

Consider the following statements:

  1. In India, credit rating agencies are regulated by the Reserve Bank of India.

  2. The rating agency popularly known as ICRA is a public limited company.

  3. Brickwork Ratings is an Indian credit rating agency.

Which of the statements given above are correct?

(a) 1 and 2 only

(b) 2 and 3 only

(c) 1 and 3 only

(d) 1, 2 and 3

Ans: b

Sub-Theme: Credit Rating

Statement 1 is incorrect: In India, credit rating agencies are regulated by Securities and Exchange Board of India (SEBI).

Statement 2 is correct: ICRA (Investment Information and Credit Rating Agency of India Ltd.) was set up in 1991 by IFCI, LIC, SBI and select banks as well as financial institutions to rate debt instruments.

Statement 3 is correct: In India there are six credit rating agencies registered under Securities and Exchange Board of India (SEBI) namely, CRISIL, ICRA, CARE, SMERA, Fitch India and Brickwork Ratings.

Credit Rating

  • A credit rating is an assessment of the creditworthiness of a borrower.
  • Individuals, corporations and governments are assigned credit ratings, whoever wants to borrow money.
  • Individuals are given ‘credit scores’, while corporations and governments receive ‘credit ratings’.
  • International credit rating agencies: Fitch Ratings, Moody’s Investors Service and Standard & Poor’s (S&P) are controlling approximately 95% of global ratings
  • In India: six credit rating agencies registered under Securities and Exchange Board of India (SEBI) namely, CRISIL, ICRA, CARE, SMERA, Fitch India and Brickwork Ratings.

 

Question 3

With reference to the Banks Board Bureau (BBB)’, which of the following statements are correct?

  1. The Governor of RBI is the Chairman of BBB.

  2. BBB recommends for the selection of heads for Public Sector Banks.

  3. BBB helps the Public Sector Banks in developing strategies and capital raising plans.

Select the correct answer using the code given below.

(a) 1 and 2 only

(b) 2 and 3 only

(c) 1 and 3 only

(d) 1, 2 and 3

Ans: b

Sub-Theme: Bank Board Bureau

Statement 1 is incorrect: Banks Board Bureau comprises the Chairman, three ex-officio members i.e. Secretary of the Department of Public Enterprises, Secretary of the Department of Financial Services and Deputy Governor of the Reserve Bank of India, and five expert members, two of which are from the private sector. The Chairman is selected by the central government and the RBI governor does not head it.

Statement 2 is correct: Banks Board Bureau recommends for the selection of head for Public Sector Banks and other key personnel if required.

Statement 3 is correct: BBB develops strategies for raising capital and improving performance of PSBs.

Banks Board Bureau

  • The Banks Board Bureau (BBB) has its genesis in the recommendations of ‘The Committee to Review Governance of Boards of Banks in India, May 2014 (Chairman J. Nayak)’.
  • It was part of the Indradhanush
  • It will make recommendations for appointment of whole-time directors as well as non-executive chairpersons of Public Sector Banks (PSBs) and state owned financial institutions.
  • The Ministry of Finance takes the final decision on the appointments in consultation with the Prime Minister’s Office.
  • Composition: Banks Board Bureau comprises the Chairman, three ex-officio members i.e. Secretary, Department of Public Enterprises, Secretary of the Department of Financial Services and Deputy Governor of the Reserve Bank of India, and five expert members, two of which are from the private
  • The Chairman is selected by the central governmnet.


 

Question 4

In India, which one of the following is responsible for maintaining price stability by controlling inflation?

(a) Department of Consumer Affairs

(b) Expenditure Management Commission

(c) Financial Stability and Development Council

(d) Reserve Bank of India

Ans: d

Sub-Theme: Monetary Policy

Option (d) is correct: In India, the responsibility for maintaining price stability and controlling inflation lies with the Reserve Bank of India (RBI). The RBI is the central bank of India and is responsible for regulating the supply of money in the economy through various monetary policy measures such as adjusting interest rates, controlling the reserve ratios of banks, and buying or selling government securities in the open market.

MONETARY POLICY

  • It is the policy under which RBI uses monetary instruments (interest rate and other instruments) under the RBI Act, 1934, to influence money supply in the economy to achieve certain macroeconomic goals.
  • Various instruments of Monetary Policy: Repo rate; Reverse Repo Rate; CRR; SLR; LAF; MSF; Bank Rate; MSS; etc.

 

Question 5

With reference to the expenditure made by an organization or a company, which of the following statements is/are correct?

  1. Acquiring new technology is capital expenditures.

  2. Debt financing is considered capital expenditure, while equity financing is considered revenue expenditures.

Select the correct answer using the code given below.

(a) 1 only

(b) 2 only

(c) Both 1 and 2

(d) Neither 1 nor 2

Ans: a

Sub-Theme: Expenditure

Statement 1 is correct: When a company funds are used to acquire, upgrade, and maintain physical assets such as property, plants, buildings, technology, or equipment are known as Capital Expenditure (CapEx) of a Company.

Statement 2 is incorrect: Equity financing is the process of raising capital through the sale of shares. It is an example of non-debt capital receipts.

Capital Expenditure (CapEx) of a Company:

  • When a company funds are used to acquire, upgrade, and maintain physical assets such as property, plants, buildings, technology, or equipment are known as Capital Expenditure (CapEx) of a Company.
  • When a company borrows money to be paid back at a future date with interest it is known as debt financing. Example: Repayment of loan is an example of capital

Equity financing:

  • Equity financing is the process of raising capital through the sale of shares. It is an example of non-debt capital receipts.
  • Capital receipts are receipts that create liabilities or reduce financial They also refer to incoming cash flows. Example: Recovery of loans and advances, disinvestment, issue of bonus shares, etc.

 

Question 6

With reference to Indian economy, consider the following statements:

  1. A share of the household financial savings goes towards government borrowings.

  2. Dated securities issued at market related rates in auctions form a large component of internal debt.

Which of the above statements is/are correct?

(a) 1 only

(b) 2 only

(c) Both 1 and 2

(d) Neither 1 nor 2

Ans: c

Sub-Theme: Basic Concept of Economy

Statement 1 is correct: The household financial savings used in part to finance government borrowing.  Governmental  securities,  often known as G-secs and Treasury Bills, are issued as a means of borrowing.

Statement 2 is correct: Over 93% of the total state debt is made up of internal debt. Internal loans, which account for the majority of the public debt, are further separated into marketable and non-marketable debt.

 

Question 7

“Rapid Financing Instrument” and “Rapid Credit Facility” are related to the provisions of lending by which one of the following?

(a) Asian Development Bank

(b) International Monetary Fund

(c) United Nations Environment Programme Finance Initiative

(d) World Bank

Ans: b

Sub-Theme: Financial Assistance

Rapid Financing Instrument (RFI) and Rapid Credit Facility (RCF) is an arm of the International Monetary Fund (IMF) which provides financial assistance to the countries in need,

  • Rapid Financing Instrument (RFI)
    • The Rapid Financing Instrument (RFI) provides rapid financial assistance, which is available to all member countries facing an urgent balance of payment needs.
    • The RFI was created as part of a broader reform to make the IMF’s financial support more flexible to address the diverse needs of member countries.
    • The RFI replaced the IMF’s previous emergency assistance policy and can be used in a wide range of circumstances.
  • Rapid Credit Facility (RCF)
    • The Rapid Credit Facility (RCF) provides rapid concessional financial assistance to low-income countries (LICs) facing an urgent Balance of Payments (BoP) need with no ex-post conditionality where a full-fledged economic program is neither necessary nor feasible.
    • The RCF was created under the Poverty Reduction and Growth Trust (PRGT) as part of a broader reform to make the Fund’s financial support more flexible and better tailored to the diverse needs of LICs, including in times of crisis.

NOTE: This was in the news due to Sri Lanka’s recent crisis and political instability. Therefore, it is very important to holistically cover the static portion and keep track of various national and international events w.r.t. UPSC syllabus.

 

Question 8

With reference to the Indian economy, consider the following statements:

  1. An increase in Nominal Effective Exchange Rate (NEER) indicates the appreciation of rupee.

  2. An increase in the Real Effective Exchange Rate (REER) indicates an improvement in trade

  3. An increasing trend in domestic inflation relative to inflation in other countries is likely to cause an increasing divergence between NEER and REER.

Which of the statements are correct?

(a) 1 and 2 only

(b) 2 and 3 only

(c) 1 and 3 only

(d) 1, 2 and 3

Ans: c

Sub-Theme: Exchange Rate

Statement 1 is correct: NEER is the Weighted average of bilateral nominal exchange rates of the home currency in terms of foreign currencies. The nominal exchange rate is the amount of domestic currency needed to purchase foreign currency. Therefore, an increase/decrease in NEER indicates the appreciation/depreciation of Rupee against the weighted basket of currencies of its trading partners.

Statement 2 is incorrect: REER is the weighted average of nominal exchange rates adjusted for relative price differential between the domestic and foreign countries. Therefore, an increase in a nation’s REER is an indication that its exports are becoming more expensive and its imports are becoming cheaper, which simply means it is losing trade competitiveness.

Statement 3 is correct: A real effective exchange rate (REER) is the NEER adjusted by relative prices or costs, typically captured in inflation differentials between the home economy and trading partners. Higher the inflation higher will be divergence (difference between) NEER and REER.

 

Question 9

Consider the following statements:

  1. Tight monetary policy of the US Federal Reserve could lead to capital flight.

  2. Capital flight may increase the interest cost of firms with existing External Commercial Borrowings (ECBs).

  3. Devaluation of domestic currency decreases the currency risk associated with ECBs.

Which of the statements given above are correct?

(a) 1 and 2 only

(b) 2 and 3 only

(c) 1 and 3 only

(d) 1, 2 and 3

Ans: a

Sub-Theme: Monetary Policy and External Commercial Borrowings

Statement 1 is correct: A tight monetary policy is implemented to contract economic growth. It involves increasing interest rates to constrain borrowing and to stimulate savings, this can discourage investment and depress asset prices. It makes borrowing less attractive as interest payment increases. Thus, tight monetary policy of the US Federal Reserve could lead to capital flight by the investors.

Statement 2 is correct: Capital flight can drive up the interest costs as there is reduced money supply in the system. Thus, it would lead to increase in the interest cost of firms that have external commercial borrowings.

Statement 3 is incorrect: ECB is basically a loan availed by an Indian entity from a non-resident lender in foreign currency whereas Devaluation is decreasing the value of currency within a fixed exchange rate system. Therefore, devaluation of the domestic currency would increase the currency risks associated with ECBs and will result in higher interest cost for borrowers.

NOTE: This is basically a conceptual based question by clubbing both the Monetary Policy and External Sector. Therefore, it underlines the importance of conceptual clarity of various economic tools, instruments, etc.

 

Question 10

With reference to the Indian economy, what are the advantages of “Inflation-Indexed Bonds (IIBs)”?

  1. Government can reduce the coupon rates on its borrowing by way of IIBs.

  2. IIBs provide protection to the investors from uncertainty regarding inflation.

  3. The interest received as well as capital gains on IIBs are not taxable.

Which of the statements given above are correct?

(a) 1 and 2 only

(b) 2 and 3 only

(c) 1 and 3 only

(d) 1, 2 and 3

Ans: a

Sub-Theme: Bonds and Securities

Statements 1 and 2 are correct: Inflation- Indexed Bonds is a debt market securities offered by the government to protect the savings from inflation and offer positive real rates of returns. The principal and interest are linked to WPI/ CPI. Inflation indexed bonds provide returns that are always in excess of inflation, ensuring that price rise does not erode the value of savings. Globally, IIBs were first issued in 1981 in the UK. In India, the Government of India through RBI issued IIBs (linked to WPI) in June 2013. They can be traded in the secondary market like other G-Secs and help the Government to reduce the coupon rates on its borrowings.

Statement 3 is incorrect: The existing tax provisions will be applicable on interest payment and capital gains on IIBs. There will be no special tax treatment for these bonds.

 

Question 11

Which of the following activities constitute a real sector in the economy?

  1. Farmers harvesting their crops

  2. Textile mills converting raw cotton into fabrics

  3. A commercial bank lending money to a trading company

  4. A corporate body issuing Rupee Denominated Bonds overseas.

Select the correct answer using the code given below.

(a) 1 and 2 only

(b) 2, 3 and 4 only

(c) 1, 3 and 4 only

(d) 1, 2, 3 and 4

Ans: a          

Sub-Theme: Sectors of Economy

The real sector of an economy is the key section as activities of this sector persuade economic output and is represented by those economic segments that are essential for the progress of GDP of the economy.

Statements 1 and 2 are correct: The sector’s (Farming and Textile Mills) ability to produce enough goods and services to satisfy global demand makes it essential for the economy’s long-term viability.

Statements 3 and 4 are incorrect: The financial sector is an area of the economy that consists of businesses and institutions that offer financial services to wholesale and retail clients. A commercial bank lending money to a trading company or a corporate body issuing rupee denominated bonds overseas constitute financial sector activities and not real sector activities.

 

 

Question 12

With reference to Convertible Bonds, consider the following statements:

  1. As there is an option to exchange the bond for equity, Convertible Bonds pay a lower rate of interest.

  2. The option to convert to equity affords the bondholder a degree of indexation to rising consumer prices.

Which of the statements given above is/are correct?

(a) 1 only

(b) 2 only

(c) Both 1 and 2

(d) Neither 1 nor 2

Ans: c         

Sub-Theme: Convertible Bond

Statement 1 is correct: A convertible bond is a fixed-income corporate debt security that yields interest payments but can be converted into a predetermined number of common stock or equity shares. Convertible bonds generally offer a lower coupon rate or rate of return in exchange for the value of the option to convert the bond into common stock. Investors will generally accept a lower coupon rate on a convertible bond, compared with the coupon rate on an otherwise identical regular bond, because of its conversion feature.

Statement 2 is correct: The option to convert to equity affords the bondholder a degree of indexation to rising consumer prices as equity prices can differ widely from the given interest and the difference in that can be used as a hedge for inflation.

 

2021

 

Question 1

Which one of the following is likely to be the most inflationary in its effects?

(a) Repayment of public debt

(b) Borrowing from the public to finance a budget deficit

(c) Borrowing from the banks to finance a budget deficit

(d) Creation of new money to finance a budget deficit

Ans: d

Sub-Theme: Inflation

Inflation is the rise in prices of goods and services within a particular economy wherein, the purchasing power of consumers decreases, and the value of the cash holdings erode. In India, the Ministry of Statistics and Programme Implementation (MoSPI) measures inflation. Some causes that lead to inflation include the creation of new money to finance a budget deficit. A Budget Deficit refers to a situation where total expenditure exceeds the total revenue.

Other Causes that lead to inflation are: Increase in demand, reduction in supply, demand-supply gap, excess circulation of money, increase in input costs, devaluation of currency, rise in wages among others.

NOTE: This question came verbatim from PYQ 2013. Therefore, it underlines the importance of solving and analysing PYQs.

 

Question 2

Which of the following steps is most likely to be taken at the time of an economic recession?

(a) Cut in tax rates accompanied by an increase in interest rate

(b) Increase in expenditure on public projects

(c) Increase in tax rates accompanied by reduction of interest rate

(d) Reduction of expenditure on public projects

Ans: b

Sub-Theme: Recession

  • A fall in the gross domestic product (GDP) for two or more consecutive quarters is often regarded as an economic recession.
  • Recessions are brought on by high interest rates because they reduce liquidity, or the quantity of money available for
  • An increase in expenditure on public projects will lead to an increase in investments, leading to an increase in GDP and income in the economy and in turn increase in demand, completing the virtuous cycle of

Additional Information:

  • Recession is a situation when an economy faces negative growth of GDP in two successive quarters. Cut in tax rates accompanied by an increase in interest rate can create a credit crunch in the economy, which will not be favourable at the moment for the economy.
  • When income is falling in the economy then an increase in tax rates accompanied by a reduction of interest rate is not desirable at the moment.
  • Reduction of expenditure on public projects in the time of recession will not be favourable as it will reduce the output of the economy.

 

Question 3

With reference to the Indian economy, demand-pull inflation can be caused/increased by which of the following?

  1. Expansionary policies

  2. Fiscal stimulus

  3. Inflation-indexing wages

  4. Higher purchasing power

  5. Rising interest rates

Select the correct answer using the code given below.

(a) 1, 2 and 4 only

(b) 3, 4 and 5 only

(c) 1, 2, 3 and 5 only

(d) 1, 2, 3, 4 and 5

Ans: a

Sub-Theme: Types of Inflation

Demand-pull inflation is caused by an increase in demand and when the demand in the economy outgrows the supply in the economy. It can be summed up as a condition of “too much money chasing too few goods”. With reference to the Indian economy, demand- pull inflation can be caused/increased by the following:

  1. Expansionary Policies: Money in the market rises when the government spends more freely. It leads to an increase in demand for the goods and fuels demand- pull inflation.
  2. Fiscal Stimulus: It also increases the money in the market which leads to an increase in demand for the goods and fuels demand- pull inflation.
  3. Inflation-Indexing Wages: It means wages are linked to the inflation which means wages move as inflation changes in the economy. Such indexing is provided to reduce the effect of inflation on wages. It can not lead to demand pull inflation in the
  4. Higher Purchasing Power: Consumers feel more confidence and spend more when they have a better income. As a result, demand increases, driving up inflation.
  5. Rising Interest Rates: It will reduce the money supply in the market. Borrowing money will become costlier, creating a credit crunch in the economy. So, it cannot cause demand to pull inflation in the economy.

Additional Information:

  • Effective change in the wages is zero and it does not increase/decrease purchasing power. So, it cannot lead to a demand to pull inflation in the economy.
  • Rising interest rates decrease the money supply in the economy. Also, due to this borrowing money becomes costly. So, it cannot cause demand-pull inflation in the

 

Question 4.

Consider the following statements:

Other things remaining unchanged, market demand for a good might increase if

(a) price of its substitute increases

(b) price of its complement increases

(c) the good is an inferior good and the income of the consumer increases

(d) its price falls

Which of the above statements are correct?

(a) 1 and 4 only

(b) 2, 3 and 4

(c) 1, 3 and 4

(d) 1, 2 and 3

Ans: a

Sub-Theme: Functions of Money Market

  • Statement 1 is correct: In the case of Substitute goods, demand for a good usually moves in the direction of the price of its substitutes. Since tea is a substitute for coffee, if the price of coffee increases, the consumers can shift to tea, and hence, the consumption of tea is likely to go up.
  • Statement 2 is incorrect: Goods that are consumed together are called Complementary goods such as tea and sugar, pen and ink etc. Since Pen and ink are used together, an increase in the price of pens is likely to reduce the demand for ink and a decrease in the price of ink is likely to increase the demand for Similar is the case with other compliments. With the increase in the price of complement, demand reduces.
  • Statement 3 is incorrect: Inferior goods are those goods demanded for which move in the opposite direction as the income of the consumer. As the income of the customer increases, the demand for inferior goods falls, and as the income decreases, the demand for them rises. E.g. coarse cereals.
  • Statement 4 is correct: As per Law of demand, When the price of the commodity decreases, demand for it rises.

 

Question 5

Consider the following statements:

1. The Governor of the Reserve bank of India (RBI) is appointed by the Central Government.

2. Certain provisions in the Constitution of India give the Central Government the right to issue directions to the RBI in public interest.

3. The Governor of the RBI draws his power from the RBI Act.

Which of the above statements are correct?

(a) 1 and 2 only

(b) 2 and 3 only

(c) 1 and 3 only

(d) 1, 2 and 3

Ans: c

Sub-Theme: Role and Functions of RBI/Banking Appointment

Statement 1 is correct: According to Section 8 of RBI ACT 1934, A Governor and (not more than four) Deputy Governors are to be appointed by the Central Government. Statement 2 is incorrect: According to Section 7 of RBI ACT 1934, The Central Government may from time to time give such directions to the Bank as it may, after consultation with the Governor of the Bank, consider it necessary in the public interest. There is no such provision in the constitution of India.

Statement 3 is correct: The Governor of the RBI draws his power from the RBI Act 1934. RESERVE BANK OF INDIA:

  • Before RBI: The functions of a central bank were virtually done by the Imperial Bank of India.
  • It was established via the RBI act 1934, and started its operations from April 1, 1935 → statutory body. Similarly, SBI is also a statutory body deriving its legality from SBI Act 1955.
  • The RBI was initially a privately held bank rather than a government-owned
  • Following independence, the government enacted the Reserve Bank (Transfer to Public Ownership) Act of 1948, which allowed it to acquire the RBI from private owners after providing fair compensation.
  • The RBI was nationalised in 1949, and as of January 1, 1949, it operated as a government-owned bank.
  • It oversees the Indian financial sector and is the top monetary and banking authority in the nation.
  • Since it holds the reserves of all commercial banks, it is known as the “Reserve Bank.”
Monetary Authority •   Formulates, implements and monitors the monetary policy.

•   Maintaining price stability while keeping in mind the objective of growth.

 

Regulator and Supervisor of the Financial System

•   Prescribes broad parameters of banking operations within which the country’s banking and financial system functions.

•   Regulation and supervision of banks under Banking Regulation Act 1949.

•   Regulation and supervision of non-banking financial companies.

•   Protecting depositors’ interest

 

Manager of Foreign Exchanges •   Manages the Foreign Exchange Management Act, 1999.

•   It facilitates external trade and payment

•   Promote development and maintenance of foreign exchange market in India.

 

Issuer of Currency

•   RBI has the sole right to issue currency notes in India.

•   Besides exchanges and destroys currency and coins not fit for circulation.

•   To give the public an adequate quantity of supplies of currency notes and coins and in good quality.

Developmental Role •   Performs a wide range of promotional functions to support national objectives such as making institutional arrangements for rural or agricultural finance.
 

Financial Inclusion

•   The Reserve Bank has selected a bank led model for financial inclusion in India.

•   RBI has undertaken a series of policy measures. E.g. Basic Savings Bank Deposit Account” (BSBDA), JAM Trinity, etc.

 

Use of Technology

•   Devices such as ATMs, handheld devices to identify user accounts through a card and biometric identifier, Deposit taking machines and Internet banking and Mobile banking facility to provide the banking services to all sections of society with more ease.
Banker to Banks •   It maintains banking accounts of all scheduled banks. It also acts as a lender of last resort by providing funds to banks.
Banker to Government •   It performs merchant banking functions for the central and the state governments.

•   It is entrusted to the central govt. ‘s money, remittances, exchange and manages its public debt as well.

 

 

 

 

 

Governor of RBI

•   Appointment: Appointed after the proposal made by the Financial Sector Regulatory Appointments Search Committee (FSRASC), headed by the Cabinet Secretary.

•   Term: According to Section 8 (4) of the RBI Act, the Governor and Deputy Governors shall hold office for such term not exceeding 3 years as the Central Government may fix when appointing them.

•   Re-Appointment: They are eligible for re-appointment.

•   Qualification: The RBI Act does not provide for any specific qualification for the governor.

•   Removal: The governor can be removed by the central government.

 

Minimum Reserve System of RBI

•   With a minimum value of government-held gold of ₹ 200 crores (₹ 115 cr rupee should be in the form of gold or gold bullion and rest ₹ 85 cr should be in the form of foreign currencies) and the remaining is backed by the government securities issued and held by RBI.
 

 

Subsidiaries of RBI

•   Deposit Insurance and Credit Guarantee Corporation (DICGC)

•   Bharatiya Reserve Bank Note Mudran Private Limited (BRBNMPL)

•   Reserve Bank Information Technology Private Ltd. (ReBIT)

•   Indian Financial Technology and Allied Services (IFTAS)

 

 

 

 

Income and Expenditure of RBI

INCOME EXPENDITURE
•   Returns from foreign currency assets

•   Interest on rupee-denominated government bonds

•   Interest on overnight lending to commercial banks

•   Management commission on handling the borrowings of central and state governments.

•   Printing of currency

•   Staff expenditure

•   Commission given to commercial banks.

•   Commission to primary dealers

 

ASSETS LIABILITIES
•   Foreign currency assets •   Currency held by Public
Assets and Liabilities of RBI •   Bill purchases and discounts

•   Collaterals by commercial banks

•   Loan and advances

•   Vault cash held by commercial banks

•   Government securities

•   Other liabilities

•   Rupee securities
•   Gold coin bullion

 

Question 6

With reference to ‘Urban Cooperative banks’ in India consider the following statements:

  1. They are supervised and regulated by local boards set up by the State Governments.

  2. They can issue equity shares and preference shares.

  3. They were brought under the purview of the Banking Regulation Act, 1949 through an Amendment in 1966.

Which of the statements given above is/are correct?

(a) 1 only

(b) 2 and 3 only

(c) 1 and 3 only

(d) 1, 2 and 3

Ans: b

Sub-Theme: Cooperative Bank

Statement 1 is incorrect: After Banking Regulation (Amendment) Act 2020 was passed, all the powers were transferred to RBI from the Registrars of the cooperative societies. Even some powers are left with the registrar but RBI powers will override them.

Statement 2 is correct: RBI has issued guidelines allowing cooperative banks to raise funds through the issuance of equity shares, preference shares and debt instruments.

Statement 3 is correct: Large cooperative banks with paid-up share capital and reserves of Rs.1 lakh were brought under the purview of the Banking Regulation Act 1949 with effect from 1st March 1966.

CO-OPERATIVE BANKS

  • A Co-operative bank belongs to its members, who are at the same time the owners and the customers of their bank.
  • Under dual control of the RBI and Registrar of Cooperative Societies.
  • The board of members are elected with each member having one vote.
  • Agriculture, some small-scale businesses, and independent contractors primarily receive funding from cooperative banks.
  • These banks are cooperative credit institutions that are registered under the Cooperative Societies Act 1912. These banks work according to the cooperative principles of mutual assistance.
  • Co-operative banks have a three-tier structure:
    • Primary Credit Societies-PCSs (agriculture or urban).
    • District Central Co-Operative Banks-
    • State Co-Operative Banks-SCBs (at the apex level).

 

Question 7

In India, the central bank’s function as the ‘lender of last resort’ usually refers to which of the following?

  1. Lending to trade and industry bodies when they fail to borrow from other sources

  2. Providing liquidity to the banks having a temporary crisis

  3. Lending to governments to finance budgetary deficits

Select the correct answer using the code given below.

(a) 1 and 2

(b) 2 only

(c) 2 and 3

(d) 3 only

Ans: b

Sub-Theme: RBI

Option (b) is correct: In India, the central bank’s function as the ‘lender of last resort’ usually refers to providing liquidity to banks that are facing temporary financial difficulties and are unable to meet their obligations. The Reserve Bank of India (RBI) can lend money to these banks, either directly or through other channels, to help them meet their short-term liquidity needs and avoid a crisis. This function is important for maintaining financial stability in the banking system and ensuring that banks can continue to serve their customers even in times of stress.

Lender of Last Resort

  • A “lender of last resort” means that the RBI (also called banker of banks) offers loans to banks or other eligible institutions that are experiencing financial difficulty or are considered highly risky or near collapse.
  • This helps to protect individuals who have deposited funds and to prevent customers from withdrawing out of panic from banks with temporarily limited liquidity.
  • Commercial banks usually try not to borrow from the “lender of last resort” e. RBI because such action indicates that the bank is experiencing a financial crisis.

 

Question 8

The money multiplier in an economy increases with which one of the following?

(a) Increase in the Cash Reserve Ratio in the banks

(b) Increase in the Statutory Liquidity Ratio in the banks

(c) Increase in the banking habit of the people

(d) Increase in the population of the country

Ans: c

Sub-Theme: Money Multiplier/Money Supply

  • The money multiplier is the amount of money created by banks for a given fixed amount of base money and reserve ratio. For example, if the commercial banks gain deposits of Rs1 Lakh and this leads to a final money supply of Rs 10 lakh. The money multiplier is 10.
  • An increase in the banking habit of the population will increase the lending, thereby will lead to more deposits in the banks hence increasing the money

 

Question 9

Which one of the following effects of the creation of black money in India has been the main cause of worry to the Government of India?

(a) Diversion of resources to the purchase of real estate and investment in luxury housing

(b) Investment in unproductive activities and purchase of precious stones, jewellery, gold,

(c) Large donations to political parties and growth of regionalism

(d) Loss of revenue to the State Exchequer due to tax evasion

Ans: d

Sub-Theme: Black Money

Option (d) is correct: The creation of black money in India causes the loss of revenue to the State Exchequer due to tax evasion, which is the main cause of worry for the Government of India.

  • Black money is the money that comes from illegal activity or unreported activity (maybe legal).
  • The first category includes everything that is earned through illegal means and the second category includes everything that is not reported to tax authorities when the income of an individual or a firm is under- Shell companies are another major source of black money coming to an economy when companies evade the tax authorities by routing their income/profits through tax havens.

 

Question 10

Consider the following statements:

The effect of the devaluation of a currency is that it necessarily:

  1. Improves the competitiveness of domestic exports in the foreign markets.

  2. Increases the foreign value of the domestic currency

  3. Improves the trade balance

Which of the above statements is/are correct?

(a) 1 only

(b) 1 and 2

(c) 3 only

(d) 2 and 3

Ans: a

Sub-Theme: BoP

Statement 1 is correct: Devaluation is decreasing the value of currency within a fixed exchange rate system. Exports become cheaper to foreign customers. Thus, it improves the competitiveness of domestic exports in the foreign markets.

Statement 2 is incorrect: Devaluation of a currency decreases the foreign value of the domestic currency. Example- Let us assume that the prevailing exchange rate of $1 is 20 Rs. So currently 1 Rs is worth $0.05. If the devaluation of currency is done and now the exchange rate of $1 is 50 Rs, this means 1 Rs is worth $0.02. So, the value of domestic currency (Rs) is decreased in terms of value of foreign currency ($).

Statement 3 is incorrect: Balance of Trade (BOT) is the difference between the value of exports and imports of a country in a given period of time. When foreign loans are valued in the native currency, devaluation also raises the debt burden on those loans. Devaluation might not, then, ultimately lead to an improvement in the trade balance. Hence, statements 3 is incorrect.

 

Question 11

Consider the following:

  1. Foreign currency convertible bonds

  2. Foreign institutional investment with certain conditions

  3. Global depository receipts

  4. Non-resident external deposits

Which of the above can be included in Foreign Direct Investments?

(a) 1, 2 and 3

(b) 3 only

(c) 2 and 4

(d) 1 and 4

Ans: a          

Sub-Theme: Foreign Investment

Statements 1, 2 & 3 are correct: Foreign Currency Convertible Bonds (FCCB), Foreign Institutional Investment with certain conditions (overall limit of 24%), and Global Depository Receipts (GDR) are the instruments for foreign investment in India.

Statement 4 is incorrect: Non-resident external deposits will create debts in the balance of payment accounts, hence not a part

of FDI.

 

Question 12

Indian Government Bond Yields are influenced by which of the following?

  1. Actions of the United States Federal Reserve

  2. Actions of the Reserve bank of India

  3. Inflation and short-term interest rates

Select the correct answer using the code given below.

(a) 1 and 2 only

(b) 2 only

(c) 3 only

(d) 1, 2 and 3

Ans: d          

Sub-Theme: Debt Instrument

Statement 1 is correct: Bond yield is the return an investor gets on that bond or on a particular government security. It depends on the price of the bond which is impacted by its demand. The actions of the US federal reserve can impact the investments flowing in India. This will lead to a decrease in demand for Government Securities (G-sec) and thus impacting its yield.

Statement 2 is correct: The actions of RBI directly impacts the bond yield because it is directly related to liquidity in the market which is controlled by RBI through various tools.

Statement 3 is correct: The purchasing capacity of an economy is directly related to inflation. So any change in short term rates will impact the demand and price of G-sec and thereby influencing the yield.

 

Question 13

With reference to India, consider the following statements:

  1. Retail investors through Demat account can invest in ‘Treasury Bills’ and ‘Government of India Debt Bonds’ in the primary market.

  2. The ‘Negotiated Dealing System-Order Matching’ is a government securities trading platform of the Reserve Bank of India.

  3. ‘Central Depository Services Ltd’ is jointly promoted by the Reserve Bank of India and the Bombay Stock Exchange.

Which of the statements given above is/are correct?

(a) 1 only

(b) 1 and 2

(c) 3 only

(d) 2 and 3

Ans: b         

Sub-Theme: Debt Instrument

Statement 1 is correct: Bond yield is the return an investor gets on that bond or on a (T-Bills) and Government bonds. It is mandatory to open a Demat account for a retail investor to invest in ‘Treasury Bills’ and ‘Government of India Debt Bonds’ in the primary market.

Statement  2  is  correct:  The  Negotiated Dealing System Order Matching is an electronic trading platform operated by the Reserve Bank of India to facilitate the issuing and exchange of government securities and other types of money market instruments.

Statement 3 is incorrect: CDSL was promoted by BSE Ltd. jointly with leading banks such as State Bank of India, Bank of India, Bank of Baroda, HDFC Bank, Standard Chartered Bank and Union Bank of India.

 

Question 14

With reference to ‘Water Credit’, consider the following statements:

  1. It puts microfinance tools to work in the water and sanitation sector.

  2. It is a global initiative launched under the aegis of the World Health Organization and the World

  3. It aims to enable poor people to meet their water needs without depending on subsidies.

Which of the statements given above are correct?

(a) 1 and 2 only

(b) 2 and 3 only

(c) 1 and 3 only

(d) 1, 2 and 3

Ans: c

Sub-Theme: Water Conservation

Statements 1 and 3 are correct: Water Credit is a powerful solution and the first to put microfinance tools to work in the water and sanitation sector. It helps bring small loans to those who need access to affordable financing and expert resources to make household water and toilet solutions a reality.

Statement 2 is incorrect: Water Credit Initiative is a loan program started by water.org to address the barrier of affordable financing for safe water and sanitation.

 

2020

 

Question 1

With reference to the Indian economy after the 1991 economic liberalisation, consider the following statements:

  1. Worker productivity (Rs. per worker at 2004 -05 prices) increased in urban areas while it decreased in rural areas.

  2. The % age share of rural areas in the workforce steadily increased.

  3. In rural areas, the growth in the non-farm economy increased.

  4. The growth rate in rural employment decreased.

Which of the statements given above is/are correct?

(a) 1 and 2 only

(b) 3 and 4 only

(c) 3 only

(d) 1, 2 and 4 only

Ans: b

Sub-Theme: Economic Reforms

“Changing Structure of Rural Economy of India: Implications for Employment and Growth 2017,” is a report published by NITI Aayog, which made the following observations:

Statement 1 is incorrect: Both rural and urban regions have seen an improvement in worker productivity as well as the absolute level of income per worker. For rural areas, it was Rs. 37273 in 2004 -05 and Rs. 101755 in 2011-12, while for urban areas it was Rs. 120419 in 2004-05 and Rs. 282515 in 2011-12.

Statement 2 is incorrect: As per the 2011 Census, 68.8% of India’s population and 72.4% of the workforce resided in rural areas. However, the steady transition to urbanisation over the years has led to a decline in the rural share of the workforce, from 77.8% in 1993-94 to 70.9% in 2011-12.

Statement 3 is correct: About two-thirds of rural income is now generated in non- agricultural activities. Non-farm economy has increased in rural areas. The share of agriculture in the rural economy has decreased from 57% in 1993-94 to 39% in 2011-12.

Statement 4 is correct: After 2004-05, the rural areas have witnessed negative growth in employment in spite of high growth in output. The growth rate of rural employment was 1.45% during 1994-2005, which fell to -0.28% between 2005-12.

1 6

 

Question 2

Which of the following factors/policies were affecting the price of rice in India in the recent past?

  1. Minimum Support Price

  2. Government’s trading

  3. Government’s stockpiling

  4. Consumer subsidies

Select the correct answer using the code given below.

(a) 1, 2 and 4 only

(b) 1, 3 and 4 only

(c) 2 and 3 only

(d) 1, 2, 3 and 4

Ans: d

Sub-Theme: Government Policies Factors/Policies Affecting the Price of Rice in recent past are:

  • Minimum Support Price: MSP is a type of market intervention that the government uses to protect farmers against a sudden drop in farm prices. Rice is included in MSP and thus impacts the price of rice.
  • Governments Trading: The Government’s Trading and Minimum Support Price of rice results in the diversion of stocks from the open market hence driving up the price for the ultimate Though factors like climatic shocks determine rice production and prices in the short run, the future scenario of rice prices must be based on long-term projections of rice demand and supply.
  • Government’s Stockpiling: Low stock levels constrain the ability to buffer the price rise resulting from other factors.
  • Consumer Subsidies: Subsidies given to consumers lower the prices for the ultimate consumer. These subsidies are generally provided by the government.

 

Question 3

With reference to chemical fertilisers in India, consider the following statements:

1. At present, the retail price of chemical fertilisers is market-driven and not administered by the

2. Ammonia, which is an input of urea, is produced from natural gas.

3. Sulphur, which is a raw material for phosphoric acid fertilizer, is a by-product of oil refineries.

Which of the statements given above is/are correct?

(a) 1 only

(b) 2 and 3 only

(c) 2 only

(d) 1, 2 and 3

Ans: b

Sub-Theme: Agriculture Fertilizer

Statement 1 is incorrect: The Union Government subsidizes fertilizers to ensure that fertilizers are easily available to farmers at reasonable prices and the county remains self- sufficient in agriculture. Thus, the fertilizer price in India is administered by the government.

Statement 2 is correct. Ammonia’s chemical formula is NH3. It is a colourless gas and is used as an industrial chemical in the production of fertilizers, plastics, synthetic fibers, dyes and other products. It occurs naturally in the environment from the breakdown of organic waste matter and may also find its way to ground and surface water sources through industrial effluents, contamination by sewage or through agricultural runoff. In the Haber– Bosch process, the atmospheric nitrogen (N2) is converted to ammonia (NH3) by reacting it with hydrogen (H2), (this Hydrogen is provided by breaking methane (CH4) from natural gas) while maintaining the high temperatures and pressures.

Statement 3 is correct. In India, the domestic production of elemental sulphur is limited to by-product recoveries from petroleum refineries and fuel oil used as feedstock for manufacturing fertilizer.

NOTE: This particular question is a multi- dimensional one that covers the aspect of Economy, Government policies, and sound knowledge in chemistry, preferably Environmental chemistry. Here, by asking this question UPSC might have wanted to check the analytical ability of aspirants from a multi-dimensional perspective.

 

Question 4

In India, which of the following can be considered as public investment in agriculture?

  1. Fixing Minimum Support Price for agricultural produce of all crops.

  2. Computerization of Primary Agricultural Credit

  3. Social Capital development.

  4. Free electricity supply to farmers.

  5. Waiver of agricultural loans by the banking system

  6. Setting up cold storage facilities by the governments.

Select the correct answer using the code given below.

(a) 1, 2 and 5 only

(b) 1, 3, 4 and 5 only

(c) 2, 3 and 6 only

(d) 1, 2, 3, 4, 5 and 6

Ans: c

Sub-Theme: Public investment in agriculture

Public Investment: It is an investment by the State (Central, state and local governments orthrough publicly owned companies) to build the nation’s capital stock by devoting resources to the basic physical infrastructure (such as roads, bridges, rail lines, airports, and water distribution), research and development, etc. that leads to increased output and/or living standards.

The following can be considered as Public Investment in Agriculture.

  • Computerization of Primary Agricultural Credit Societies: Computerization of Primary Agricultural Credit Societies will enhance productivity in the agricultural sector, as there will be easy and timely access of credit.
  • Social Capital Development: Building social capital for smallholders can also positively impact the adoption of new technologies such as the use of improved seeds, soil and water conservation practices, and agroforestry.
  • Setting up cold storage facilities by the governments: Setting up cold storage facilities will enhance productivity in the agricultural sector as agricultural products are generally perishable in nature and the facility of cold storage may help in increasing the shelf life of the products.

Additional Information:

  • Fixing Minimum Support Price for Agriculture for agricultural products of all crops and free electricity supply to farmers are subsidies, hence they are not categorized as public investment in agriculture.
  • Subsidies and incentives are not considered as public investment.

 

Question 5

Under the Kisan Credit Card scheme, short-term credit support is given to farmers for which of the following purposes?

  1. Working capital for maintenance of farm assets

  2. Purchase of combine harvesters, tractors and mini trucks.

  3. Consumption requirements of farm households

  4. Post-harvest expense

  5. Construction of a family house and setting up a village cold storage facility.

Select the correct answer using the code given below:

(a) 1, 2 and 5 only

(b) 1, 3 and 4 only

(c) 2, 3, 4 and 5 only

(d) 1, 2, 3 4 and 5

Ans: b

Sub-Theme: Agricultural credit

Announced in the 1998–1999 budget, the Kisan Credit Card Scheme aims to provide farmers with the institutional credit they need to meet their financial needs at various phases of farming. It is implemented by all public sector banks, regional rural banks, and cooperative banks across the nation. Under the Kisan Credit Card scheme, short-term credit support is given to farmers for the following purposes:

  • Working Capital for Maintenance of Farm Assets: To provide Working capital for maintenance of farm assets and activities allied to agriculture.
  • Consumption Requirement of Farm Households: To fulfill the consumption requirements of farmers is one of the main objectives of KCS.
  • Post Harvest Expenses: Supporting the farmer in their post-harvest expenses.

Additional Information:

  • The Kisan Credit Cards are issued to the farmers so that they may use them to purchase agriculture inputs such as seeds, fertilizers, pesticides etc. and draw cash for their production needs.
  • Construction of a family house is covered under PMAY and setting up a village cold storage facility is not covered under KCC

NOTE: Statement 2 talks about “Purchase of combine harvesters, tractors and mini trucks” which is not a short term investment. It needs heavy capital and is not possible with short-term credit support provided under KCC. By this reasoning and general understanding if we eliminate statement 2, we will get correct answer i.e., ‘Option (b) 1, 3 and 4 only’

 

Question 6.

Consider the following statements:

1. In the case of all cereals, pulses and oil-seeds, the procurement at Minimum Support Price (MSP) is unlimited in any State/UT of India.

2. In the case of cereals and pulses, the MSP is fixed in any State/UT at a level to which the market price will never rise.

Which of the statements given above is/are correct?

(a) 1 only

(b) 2 only

(c) Both 1 and 2

(d) Neither 1 nor 2

Ans: d

Sub-Theme: MSP

Statement 1 is incorrect: Procurement of all cereals, pulses and oilseeds at Minimum support price is open-ended. It is limited since our buffer stock is limited. Open ended procurement means whatever foodgrains are offered by the farmers, within the stipulated procurement period and which conforms to the quality specifications prescribed by Government of India, are purchased at MSP (and bonus/ incentive, if any) by the Government agencies including FCI for central Pool.

Statement 2 is incorrect: Market price has no link with minimum support price and it can go below or above the MSP, depending upon the demand of the crop in the market.

 

Question 7

The term ‘West Texas Intermediate’, sometimes found in news, refers to a grade of

(a) Crude oil

(b) Bullion

(c) Rare earth elements

(d) Uranium

Ans: a

Sub-Theme: Crude oil Trade

Crude Oil Trade:

  • There are about 160 crude oils that are traded internationally.
  • They vary in terms of their characteristics & quality.
  • The two main benchmarks for crude oil used internationally are West Texas Intermediate and Brent Crude.

Difference of WTI and Brent Crude oil:

WTI Brent
Benchmark for oil extracted from America Benchmark for crude oil obtained from

the North Sea near Norway, Sweden, and UK

Benchmark used by US oil prices Benchmark used for OPEC oil prices
Traded on New York Mercantile Exchange Traded on International Exchange in London
Low share at international trade but futuristic opportunities Two-third of the world’s crude contracts are signed Brent oil Benchmark

 

Question 8

With reference to Trade-Related Investment Measures (TRIMS), which of the following statements is/are correct?

  1. Quantitative restrictions on imports by foreign investors are prohibited.

  2. They apply to investment measures related to trade in both goods and services.

  3. They are not concerned with the regulation of foreign investment.

Select the correct answer using the code given below:

(a) 1 and 2 only

(b) 2 only

(c) 1 and 3 only

(d) 1, 2 and 3

Ans: c

Sub-Theme: WTO Agreements

Statement 1 is correct: Trade-Related Investment Measure (TRIMs) provides quantitative restrictions on imports by foreign investors. TRIMs agreement stipulates that certain measures adopted by Governments to regulate FDI can cause trade-restrictive and distorting effects.

Statement 2 is incorrect: As per Article 1 of the TRIMs agreement, it applies only to investment measures related to trade in goods and not in services.

Statement 3 is correct: TRIMs are not intended to deal with the regulation of investment as such and does not impact directly on WTO members’ ability to regulate and place conditions upon the entry and establishment of foreign investment.

 

Question 9

Consider the following statements:

1. The weightage of food in Consumer Price Index (CPI) is higher than that in Wholesale Price Index (WPI).

2. The WPI does not capture changes in the prices of services, which CPI does.

3. The Reserve Bank of India has now adopted WPI as its key measure of inflation and to decide on changing the key policy rates.

Which of the statements given above is/are correct?

(a) 1 and 2 only

(b) 2 only

(c) 3 only

(d) 1, 2 and 3

Ans: a

Sub-Theme: Inflation

Statement 1 is correct: The Wholesale Price Index (WPI) measures wholesale price changes. The Wholesale goods or services sold by businesses to smaller businesses for selling further. The WPI does not include services. The weightage for food is lower in WPI whereas the weightage of food in the CPI is close to 50%.

Statement 2 is correct: WPI indicates the wholesale price, whereas CPI shows the retail price i.e., the price at which people make purchases from the retail market. WPI does not show the impact of inflation on the people. Any policy should consider the impact on the people. WPI does not account for the price of services. It is an international best practice – most of the countries have shifted to CPI.

Statement 3 is incorrect. The Reserve bank of India adopted the new Consumer Price Index (CPI) (combined) as the key measure of inflation in 2014. The Consumer Price Index measures retail-level price changes. It includes services as well.

 

Question 10

If another global financial crisis happens in the near future, which of the following actions/policies are most likely to give some immunity to India?

  1. Not depending on short-term foreign borrowings

  2. Opening up to more foreign banks

  3. Maintaining full capital account convertibility

Select the correct answer using the code given below:

(a) 1 only

(b) 1 and 2 only

(c) 3 only

(d) 1, 2 and 3

Ans: a

Sub-Theme: Mechanism to counter Financial crisis

Statement 1 is correct: Due to the global economic crisis, both creditor and debtor nations face hardships to keep the economy afloat. Various  political and economic considerations make it difficult to come to the rescue of a distressed nation. Hence India should not depend on short term borrowings.

Statement 2 is incorrect: Opening up to more foreign banks would lead to enhanced exposure to the global economy, and hence an increased risk. Strengthening the domestic banks would prove more helpful in a situation like a global economic crisis.

Statement 3 is incorrect: Capital account convertibility means no restriction on the amount you can convert into foreign currency to enable you to acquire any foreign assets and vice versa. In a situation of a financial crisis, it will be a big mistake. It can create a situation of “Capital flight” where a foreign investor can withdraw all his money at once.

 

Question 11

If you withdraw Rs. 1,00,000 in cash from your Demand Deposit Account at your bank, the immediate effect on aggregate money supply in the economy will be:

(a) To reduce it by 1,00,000

(b) To increase it by 1,00,000

(c) To increase it by more than 1,00,000

(d) To leave it unchanged

Ans: d

Sub-Theme: Demand and Supply of Money

Demand Deposit

  • Demand deposit accounts consist of funds held in a bank account from which deposited funds can be withdrawn at any time while a term deposit account restricts access for a predetermined time (Fixed deposits accounts, Recurring deposits accounts). Current accounts and savings accounts are demand deposits.
  • Money = total currency with the public + demand deposits of the public with banks. In this given situation when you withdraw Rs. 1,00,000 from the bank, it goes to the currency in hand from demand deposits in banks but it does not change the value of the money supply.

 

Question 12

What is the importance of the term “Interest Coverage Ratio” of a firm in India?

1. It helps in understanding the present risk of a firm that a bank is going to give a loan to.

2. It helps in evaluating the emerging risk of a firm that a bank is going to give a loan to.

3. The higher a borrowing firm’s level of Interest Coverage Ratio, the worse is its ability to service its debt.

Select the correct answer using the code given below.

(a) 1 and 2 only

(b) 2 only

(c) 1 and 3 only

(d) 1, 2 and 3

Ans: a

Sub-Theme: Banking Interest

Statements 1 and 2 are correct: The interest coverage ratio is a debt and profitability ratio used to determine how easily a company can pay interest on its outstanding debt.

Lenders, investors, and creditors often use this formula to determine a company’s riskiness relative to its current debt or for future borrowing.

Statement 3 is incorrect: The higher the coverage ratio, the easier it should be to make interest payments on its debt or pay dividends. Interest Coverage Ratio:

  • The interest coverage ratio is a debt and profitability ratio used to determine how easily a company can pay interest on its outstanding debt.
  • The interest coverage ratio is calculated by dividing a company’s earnings before interest and taxes (EBIT) by its interest expense during a given period. Lenders, investors, and creditors often use this formula to determine a company’s riskiness relative to its current debt or for future
  • A coverage ratio, broadly, is a metric intended to measure a company’s ability to service its debt and meet its financial obligations, such as interest payments or dividends.
  • The higher the coverage ratio, the easier it should be to make interest payments on its debt or pay dividends.

 

Question 13

If the RBI decides to adopt an expansionist monetary policy, which of the following would it not do?

  1. Cut and optimise the Statutory Liquidity Ratio

  2. Increase the Marginal Standing Facility Rate

  3. Cut the Bank Rate and Repo Rate

Select the correct answer using the code given below:

(a) 1 and 2 only

(b) 2 only

(c) 1 and 3 only

(d) 1, 2 and 3

Ans: b

Sub-Theme: Monetary policy

Statement 1 is incorrect: Reducing SLR leaves more liquidity with banks, which in turn can fuel growth and demand in the economy.

Statement 2 is correct: With the increase of MSF Rate, cost of borrowing increases for banks resulting in reduced available resources to lend.

Statement 3 is incorrect: Under expansionary monetary policy, RBI reduces repo rate and bank rate to increase liquidity in the banking sector.

RBI Monetary Policy:

  • When the government reduces spending and raises taxes simultaneously in the nation, it is said to be implementing a contractionary fiscal policy, but when the government increases expenditure and lowers tax rates to spur economic growth, it is said to be implementing an expansionary fiscal
  • Contractionary policy slows the rate of growth in the money supply to control
  • When the government reduces spending and raises taxes simultaneously in the nation, it is said to be implementing a contractionary fiscal policy, but when the government increases expenditure and lowers tax rates to spur economic growth, it is said to be implementing an expansionary fiscal
Tool Contractionary Policy Expansionary Policy
Cash Reserve Ratio (CRR) Increase Decrease
Repo Rate Increase Decrease
Statutory Liquidity Ratio (SLR) Increase Decrease
Marginal Standing Facility Rate (MSF) Increase Decrease

 

Question 14

Consider the following statements:

1. In terms of short-term credit delivery to the agriculture sector, District Central Cooperative Banks (DCCBs) deliver more credit in comparison to Scheduled Commercial Banks and Regional Rural Banks.

2. One of the most important functions of DCCBs is to provide funds to the Primary Agriculture Credit societies.

Which of the statements given above is/are correct?

(a) 1 only

(b) 2 only

(c) Both 1 and 2

(d) Neither 1 nor 2

Ans: b

Sub-Theme: Cooperative Banking

Statement 1 is incorrect: Although the focus of rural cooperative lending is agriculture, the share in credit flow to the agriculture of rural cooperatives is only 12.1%, as compared to 76% of Scheduled Commercial Banks (SCBs), and 11.9% of Regional Rural Banks.

Statement 2 is correct: DCCBs mobilise deposits from the public and provide credit to the public and PACS.

District Central Co-operative Banks (DCCBs):

  • Rural co-operatives include District Central Co-operative Banks (DCCBs), State Co-operative Banks (StCBs), and Primary Agricultural Credit Societies (PACS).

3-tier structure  of Short  term Co-operative Banks:

  1. State Co-operative Banks
  2. District Central Co-operative Banks
  3. Primary Agricultural Credit Societies.
    • Although the focus of rural cooperative lending is agriculture, the share in credit flow to the agriculture of rural cooperatives is only 12.1%, as compared to 76% of Scheduled Commercial Banks (SCBs), and 11.9% of Regional Rural Banks.
    • DCCBs mobilise deposits from the public and provide credit to the public and PACS.

 

Question 15

In the context of the Indian economy, non-financial debt includes which of the following?

  1. Housing loans owed by households

  2. Amounts outstanding on credit cards

  3. Treasury bills

Select the correct answer using the code given below:

(a) 1 only

(b) 1 and 2 only

(c) 3 only

(d) 1, 2 and 3

Ans: d

Sub-Theme: Debt Financing

Option (d) is correct: Types of non-financial debt are Housing loans owed by households, Amounts outstanding on credit cards, Treasury bills, Credit Card balance etc.

Debt Financing:

  • When an economy borrows money to be paid back at a future date along with interest, it is known as Debt Financing.
  • Debt financing mainly includes two types of debt:
  1. Public Debt:
    • It is the total amount borrowed by the government of a country.
    • It is further divided into two types:
  2. Internal Public Debt
  3. External Public Debt
  4. Non-Financial Debt:
    • It is made up of credit instruments that are not included in the financial sector and are issued by businesses, consumers, and governments.
    • Types of non-financial debt: Housing loans owed by households, Amounts outstanding on credit cards, Treasury bills, Credit Card balance etc.

 

Question 16

With reference to the international trade of India at present, which of the following statements is/are correct?

1. India’s merchandise exports are less than its merchandise imports.

2. India’s imports of iron and steel, chemicals, fertilisers and machinery have decreased in recent years.

3. India’s exports of services are more than its imports of services.

4. India suffers from an overall trade/current account deficit.

Select the correct answer using the code given below:

(a) 1 and 2 only

(b) 2 and 4 only

(c) 3 only

(d) 1, 3 and 4 only

Ans: d         

Sub-Theme: BoP

Statement 1 is correct: The major portion of India’s current account deficit is in the area of merchandise trade. As per RBI’s data, India’s Merchandise exports during April-August 2019- 2020 was USD 133.14 billion, as compared to USD 210.39 billion of imports during the same period.

Statement 2 is incorrect: Between 2011–12 and 2018–19, the composition of imports by commodity reveals that imports of iron and steel, organic chemicals, and industrial machinery have experienced positive growth rates as a percentage of total imports.

Statement 3 is correct: India has a surplus of net services (service exports minus service imports). India’s Service exports during April- August 2019- 2020 was USD 67.24 billion, as compared to USD 39.25 billion of imports during the same period.

Statement 4 is correct: Current Account Deficit (CAD) or trade deficit is the shortfall between exports and imports. As per Economic Survey 2019-20, India’s CAD was 2.1% in 2018-19, and 1.5% of GDP in H1 of 2019-20.

 

Question 17

With reference to Foreign Direct Investment in India, which one of the following is considered its major characteristic?

(a) It is the investment through capital instruments essentially in a listed company.

(b) It is a largely non-debt creating capital flow.

(c) It is an investment that involves debt-servicing.

(d) It is the investment made by foreign institutional investors in the Government Securities.

Ans: b

Sub-Theme: Foreign Investment – FDI

Option (a) is incorrect: FDI is the investment through a capital instrument by a non- resident entity/person resident outside India in an unlisted Indian company, or 10% or more of the post issue paid-up equity capital on a fully diluted basis of a listed Indian company.

Option (b) is correct: A non-debt creating capital flow is the one where there is no direct repayment obligation for the residents. FDI is largely a non-debt creating capital flow.

Option (c) is incorrect: Debt servicing is the regular repayment of interest and principal on a debt for a particular period. FDI has no link with this concept.

Option (d) is incorrect: The investment can be made in equities or equity linked instruments or debt instruments issued by the company. Thus, FDI isn’t directly associated with government securities.

 

Question 18

“Gold Tranche” (Reserve Tranche) refers to:

(a) A loan system of the World Bank

(b) One of the operations of a Central Bank

(c) A credit system granted by WTO to its members

(d) A credit system granted by IMF to its members

Ans: d

Sub-Theme: Functions of IMF

Option (d) is correct: A Reserve Tranche is a portion of the required quota of currency each member country must provide to the IMF that can be utilized for its own purposes without a service fee or economic reform conditions.

Gold or Reserve Tranche:

  • The International Monetary Fund (IMF) is funded through its members and their quota contributions.
  • A reserve tranche is a component of the required quota of currency that each member nation must contribute to the IMF and can be used for its own needs free of charges or requirements for economic reform.
  • Essentially, the reserve tranche serves as an emergency account that IMF members can use whenever necessary.
  • The reserve tranches that nations have with the IMF are regarded as their facilities of last resort, which means they will use the reserve tranche urgently before requesting a formal credit tranche.

There are no interest costs for the initial 25% reserve tranche. Any additional items would incur a service charge.

 

Question 19

With reference to the Indian economy, consider the following statements:

1. ‘Commercial Paper’ is a short-term unsecured promissory note.

2. ‘Certificate of Deposit’ is a long-term instrument issued by the Reserve Bank of India to a

3. ‘Call Money’ is a short term finance used for interbank transactions.

4. ‘Zero-Coupon Bonds are the interest-bearing short term bonds issued by the Scheduled Commercial Banks to corporations.

Which of the statements given above is/are correct?

(a) 1 and 2 only

(b) 4 only

(c) 1 and 3 only

(d) 2, 3 and 4 only

Ans: c

Sub-Theme: Instruments of Money Market

Statement 1 is correct: Commercial Paper is a money market instrument for financing working capital requirements of companies. It is an unsecured instrument issued in the form of promissory notes which can be issued for a period ranging from 15 days to one year.

Statement 2 is incorrect: Certificate of Deposit are short term money market instruments issued by Commercial Banks and special financial institutions which are freely transferable between parties. Its maturity period ranges from 91 days to 1 year. These can be issued to individuals, cooperatives and companies.

Statement 3 is correct: Call money is a money market instrument used by the banks to meet their temporary requirement of cash. They borrow and lend money from each other normally on a daily basis. It has a maturity period of one day to fifteen days and is used by banks for adjusting to their short-term liquidity imbalances.

Statement 4 is incorrect: Zero coupon Bond is a type of bond which is issued at a discount to its face value, at which it will be redeemed. There are no intermittent payments of interests and they are generally issued for long tenure.

 

2019

 

Question 1

With reference to India’s Five -Year Plans, which of the following statements is/are correct?

1. From the Second Five -Year Plan, there was a determined thrust towards substitution of basic and capital good industries.

2. The Fourth Five -Year Plan adopted the objective of correcting the earlier trend of increased concentration of wealth and economic power.

3. In the Fifth Five -Year Plan, for the first time, the financial sector was included as an integral part of the Plan.

Select the correct answer using the code given below.

(a) 1 and 2 only

(b) 2 only

(c) 3 only

(d) 1, 2 and 3

Ans: a

Sub-Theme: Economic Planning

Statement 1 is correct: From the Second Five Year Plan (1956-61), there was determined thrust towards substitution of basic and capital goods industries. This plan focused on Rapid Industrialization with an emphasis on capital goods and heavy industries. It was based on the PC Mahalanobis Model.

Statement 2 is correct: The Fourth Plan (1969-74) provided a necessary corrective to the earlier trend which helped particularly the stronger sections in agriculture as well as in industry to enable them rapidly to enlarge and diversify the production base. It has adopted the objective of correcting the earlier trend of increased concentration of wealth and economic power. Thus, it targeted Growth with Stability and progressive achievement of self- reliance.

Statement 3 is incorrect: The Fifth plan (1974-78) aimed at removal of poverty and attaining self-reliance. The financial sector became an integral part of the plan in the Ninth five -year plan (1997-2002).

 

Question 2

The economic cost of food grains to the Food Corporation of India is Minimum Support Price and bonus (if any) paid to the farmers plus:

(a) Transportation cost

(b) Interest cost only

(c) Procurement incidentals and distribution

(d) Procurement incidentals and charges for godowns.

Ans: c

Sub-Theme: Government Policies/Food Security

Food Corporation of India (FCI):

  • Food Corporation of India (FCI) is a Public Sector Undertaking that falls under the Ministry of Consumer Affairs’ Department of Food and Public Distribution.
  • In 1965, the Food Corporations Act of 1964 created the FCI, a statutory body. It was founded in the midst of a severe grain crisis, particularly in wheat.
  • In order to suggest remunerative pricing to farmers, the Commission on Agricultural

Costs and Prices (CACP) was established in 1965.

  • The Economic Cost of food grains to the FCI is the Minimum Support Price and bonus (if any) paid to the farmers plus the procurement incidentals and distribution
  • Procurement incidentals are expenses incurred during procurement till the food grains reach the first point of godown.
  • Distribution costs are now included in economic costs, whilst buffer carrying costs are now included in buffer subsidies.

 

Question 3

Among the agricultural commodities imported by India, which one of the following accounts for the highest imports in terms of value in the last five years?

(a) Spices

(b) Fresh fruits

(c) Pulses

(d) Vegetable oils

Ans: d

Sub-Theme: Agricultural commodity Trading

Import of Agricultural commodities:

  • Vegetable oils account for the highest import in terms of value in the last five years. India relies on imports for 70% of its edible oil consumption.
  • Pulses account for second-most highest while fresh fruits account for third highest and spices account for least imports in terms of value in the last five years among the above-mentioned agricultural commodities imported in India.

 

Question 4

With reference to land reforms in independent India, which one of the following statements is correct?

(a) The ceiling laws were aimed at family holdings and not individual holdings.

(b) The major aim of land reforms was providing agricultural land to all the landless.

(c) It resulted in cultivation of cash crops as a predominant form of cultivation.

(d) Land reforms permitted no exemptions to the ceiling limits.

Ans: b

Sub-Theme: Land Reforms

Land Reforms in Independent India:

  • The Land Reforms in India aimed at the redistribution of ownership holdings and reorganizing operational holdings from the view point of optimum utilization of land.
  • It was a major aim of land reforms to provide agricultural land to all the landless.
  • The Objectives of the Land Reforms are as follows:
    • Removing institutional discrepancies of the agrarian structure.
    • Providing agricultural land to all the landless.
    • Issues of socio-economic inequality.
    • In order to address the interconnected issues of poverty, hunger, and food insecurity, agricultural productivity must be increased.

Additional Information:

  • Land Ceiling laws were applied to family holdings as well as individual holdings depending upon the region/state.
  • The outcome of Land reforms: (i) abolition of intermediaries, (ii) fixation of ceilings on land holdings, and (iii) redistribution of surplus land among landless or semi-landless peasants. Cultivation of cash crops requires much more other inputs rather than the mere availability of holdings. Commercialization of agriculture, and not land reforms, led to an increase in the cultivation of cash crops.
  • Certain exemptions to the ceiling limits were allowed for plantations of crops like tea and coffee.

 

Question 5

Among the following, which one is the largest exporter of rice in the world in the last five years?

(a) China

(b) India

(c) Myanmar

(d) Vietnam

Ans: b

Sub-Theme: Agricultural commodity Trading

In recent years (2014-18), Vietnam’s rice exports were between 13 and 16 percent, Thailand’s between 22 and 25 percent, and India’s between 25 and 26 percent. Since the beginning of this decade, India has been the leading exporter of rice worldwide (2011-12).

 

Question 6

Consider the following statements:

1. Coal sector was nationalised by the Government of India under Indira Gandhi.

2. Now, coal blocks are allocated on lottery basis.

3. Till recently, India imported coal to meet the shortages of domestic supply, but now India is self-sufficient in coal products.

Which of the statements given above is/are correct?

(a) 1 only

(b) 2 and 3 only

(c) 3 only

(d) 1, 2 and 3

Ans: a

Sub-Theme: Coal Sector

Statement 1 is correct: The nationalisation was done in two phases, the first with the coking coal mines in 1971-72 and then with the non -coking coal mines in 1973. Indira Gandhi served as the Prime Minister of the country from 1971-1973.

Statement 2 is incorrect: After the Supreme Court cancelled the coal block allocations in 2014. To manage and reallocate the cancelled blocks in a transparent and accountable manner, the Coal Mines (Special Provisions) Act, 2015 enabled the provisions for ‘allocation of coal mines by way of auction and allotment for the sale of coal’.

Statement 3 is incorrect: India has the 5th largest coal reserves in the world, but due to the monopolies of some firms, it cannot produce enough coal to meet the shortages. It imports coal to meet the shortages. Domestic coal has been able to meet only 75% of our annual coal demand. About 200 mt of imports are used to make up for the shortage of local coal supply.

NOTE: Even if we don’t know the exact answer, we can still attempt the questions through elimination method. For example, observing the third statement, ‘self sufficient in coal products’ sounds absurd. Now if we have covered daily newspapers and NCERTs (geography) then we know that India holds the 5th biggest coal reserves in the world but due to incapacity of coal production by monopolistic firms, it imports coal to meet the shortages of domestic supply.

Hence, statement 3 is wrong and by eliminating this single statement we can arrive at the correct answer.

 

Question 7

The Service Area Approach was implemented under the purview of

(a) Integrated Rural Development Programme

(b) Lead Bank Scheme

(c) Mahatma Gandhi National Rural Employment Guarantee Scheme

(d) National Skill Development Mission

Ans: b

Sub-Theme: Government Policies/Financial Inclusion

Option (b) is correct: Service Area Approach is a developed version of the “area approach” method of the Lead Bank Scheme.

Service Area Approach (SAA):

  • It is a developed version of the “area approach” method of the Lead Bank
  • Under SAA, each commercial bank branch in the rural and semi-urban area was designated to serve 15 to 25 villages for the planned and orderly development of the areas and the concerned branch was responsible for meeting the needs of bank credit of its service area.
  • The service area approach was introduced in 1989 as part of the Lead Bank Scheme.
  • The main goals of SAA were to boost lending that was productive and create solid connections between bank credit, production, productivity, and rising income

 

Question 8

The Chairman of public sector banks are selected by the:

(a) Banks Board Bureau

(b) Reserve Bank of India

(c) Union Ministry of Finance

(d) Management of concerned bank

Ans: (a)

Sub-Theme: PSU Bank/Banking Appointment

Option (a) is correct: Banks Board Bureau recommends for selection of heads – Public Sector Banks and Financial Institutions and helps banks in developing strategies and capital raising plans.

Bank Board Bureau

  • It was established as an independent organisation in February 2016 based on the suggestions of the RBI appointed Nayak
  • It was part of the Indradhanush
  • It suggested for the selection of full-time directors and non-executive chairs for Public Sector Banks (PSBs) and state-owned financial institutions.
  • In cooperation with the Prime Minister’s Office, the Ministry of Finance makes the final decision regarding the appointments.

Composition:

  • Banks Board Bureau comprises the Chairman, three ex-officio members i.e., Secretary, Department of Public Enterprises, Secretary of the Department of Financial Services and Deputy Governor of the Reserve Bank of India, and five expert members, two of which are from the private sector.

 

Question 9

Which of the following is not included in the assets of a commercial bank in India?

(a) Advances

(b) Deposits

(c) Investments

(d) Money at call and short notice

Ans: b

Sub-Theme: Banking mechanism

Bank Assets:

  • A bank places its funds in assets to earn
  • The assets include investments, money at call and short notice, loans and advances and bills discounted and purchased.
  • Additionally, it covers the cash in hand with the banks and also the cash held with the RBI.
  • The liabilities include deposits (both time and demand) and borrowings.

 

Question 10

Which of the following is issued by registered foreign portfolio investors to overseas investors who want to be part of the Indian stock market without registering themselves directly?

(a) Certificate of Deposits

(b) Commercial Paper

(c) Promissory Note

(d) Participatory Note

Ans: d

Sub-Theme: Foreign Investment

P-Notes or Participatory Notes

  • These are overseas derivative instruments with underlying assets that are Indian
    • They permit unregistered overseas investors to purchase equities listed on Indian exchanges.

 

Question 11

Which one of the following is not the most likely measure the Government/ RBI takes to stop the slide of the Indian rupee?

(a) Curbing imports of nonessential goods-and promoting exports

(b) Encouraging Indian borrowers to issue rupee denominated Masala Bonds

(c) Easing conditions relating to external commercial borrowing

(d) Following an expansionary monetary policy

Ans: d

Sub-Theme: RBI monetary policy

Option (a) is incorrect: This would help control imports and thus the depreciation of the rupee.

Option (b) is incorrect: Masala bonds were brought in to curb the slide of rupee since the borrowing is rupee-dominated and does not put pressure on our currency through borrowing dollars.

Option (c) is incorrect: Easing ECBs will lead to higher borrowing abroad and would temporarily bridge the deficit of forex in India preventing the slide of rupee.

Option (d) is correct: An expansionary monetary policy may lead to lower interest rates and thus flight of foreign capital from India (which would get better returns abroad). Also, such a policy may fuel inflation and higher imports through higher government spending and further cause a slide of the rupee.

 

Question 12

The money multiplier in an economy increases with which one of the following?

(a) Increase in the Cash Reserve Ratio in the banks

(b) Increase in the Statutory Liquidity Ratio in the banks

(c) Increase in the banking habit of the people

(d) Increase in the population of the country

Ans: (c)

Sub-Theme: Money Multiplier/Money Supply

  • The money multiplier is the amount of money created by banks for a given fixed amount of base money and reserve ratio. For example, if the commercial banks gain deposits of Rs1 Lakh and this leads to a final money supply of Rs 10 lakh. The money multiplier is 10.
  • An increase in the banking habit of the population will increase the lending, thereby will lead to more deposits in the banks hence increasing the money

 

Question 13

In the context of India, which of the following factors is/are contributor/contributors to reducing the risk of a currency crisis?

  1. The foreign currency earnings of India’s IT sector

  2. Increasing the government expenditure

  3. Remittances from Indians abroad

Select the correct answer using the code given below.

(a) 1 only

(b) 1 and 3 only

(c) 2 only

(d) 1, 2 and 3

Ans: b         

Sub-Theme: Currency crisis/BoP

Currency crisis is brought about by a decline in the value of a country’s currency which negatively affects an economy. It creates instabilities in exchange rates, meaning that one unit of a certain currency no longer buys as much as it used to in another currency.

Statements 1 and 3 are correct: The foreign current earnings of India’s IT sector and remittances from abroad would lead to more inflow of foreign currencies in the economy and boost the foreign exchange reserves.

Statement 2 is incorrect: Increasing government expenditure will have no effect on the value of the currency since it is not related to change in foreign exchange reserves or any currency fluctuations.

 

Question 14

Consider the following statements:

1. Most of India’s external debt is owed by governmental entities.

2. All of India’s external debt is denominated in US dollars.

Which of the statements given above is/are correct?

(a) 1 only

(b) 2 only

(c) Both 1 and 2

(d) Neither 1 nor 2

Ans: d          

Sub-Theme: External Debt

Statement 1 is incorrect: As per the Government report on External debt, December 2018-Commercial borrowings are the largest component of external debt with a share of 37.1%, followed by NRI deposits (23.9%) and short-term trade credit (19.9%).

Statement 2 is incorrect: US dollar- denominated debt continued to be the largest component of India’s external debt with a share of 45.9 % at the end of December 2018, followed by the Indian Rupee (24.8 %), SDR (5.1 %), Yen (4.9 %) and Euro (3.1 %).

 

Question 15

Consider the following statements:

1. Purchasing Power Parity (PPP) exchange rates are calculated by comparing the prices of the same basket of goods and services in different

2. In terms of PPP dollars, India is the sixth largest economy in the world.

Which of the statements given above is/are correct?

(a) 1 only

(b) 2 only

(c) Both 1 and 2

(d) Neither 1 nor 2

Ans: a         

Sub-Theme: Purchasing Power

Statement 1 is correct: PPP is an economic theory that compares different countries’ currencies through a “basket of goods” approach. The PPP between two countries measures the amount of one country’s currency required to purchase a basket of goods and services in the country that is compared to the amount of another country’s currency in order to purchase a similar basket of goods and services. e.g. -If a dumpling is selling in India for Rs100 and in China for ¥2, this would imply a PPP exchange rate of 1¥ to Rs 50.

Statement 2 is incorrect: In terms of PPP dollars. China is the world’s largest economy, followed by the US and India at third position (2018) respectively.

 

Question 16

In a given year in India, official poverty lines are higher in some States than in others because:

(a) Poverty rates vary from State to State

(b) Price levels vary from State to State

(c) Gross State Product varies from State to State

(d) Quality of public distribution varies from State to State

Ans: b

Sub-Theme: Poverty

  • Poverty Lines: Poverty is pronounced deprivation in well-being and comprises many dimensions. It includes low incomes and the inability to acquire the basic goods and services necessary for survival with dignity – World Bank.
  • Poverty lines would vary from State to State because of inter-state price
  • According to the Planning Commission, the national poverty line for rural regions in 2011–12 was estimated to be Rs. 816 per capita per month in rural areas and Rs. 1,000 per capita per month in urban areas using the Tendulkar methodology.
  • The population’s income distribution and the basis for consumer spending determine the poverty line.
  • The poverty line is a function of the cost of the consumption basket which varies from state to state.

 

2018

Question 1

Increase in absolute and per capita real GNP does not connote a higher level of economic development, if:

(a) Industrial output fails to keep pace with agricultural output.

(b) Agricultural output fails to keep pace with industrial output.

(c) Poverty and unemployment increase.

(d) Imports grow faster than exports.

Ans: c

Sub-Theme: Economic Growth

Option (c) is correct: It is possible that an increase in absolute and per capita Gross National Product (GNP) will only reflect the economic growth of a small portion of society, leaving the rest of the population in poverty and unemployment. This is because the concept of per capita real income or per capita GDP are not able to capture the true picture in terms of non-monetary social indicators. Thus, an increase in absolute and per capita real GNP does not connote a higher level of economic development, if poverty and unemployment increase.

Additional Information: Economic Growth and Economic Development:

  • Economic Growth: A rise in real national income or national output is known as Economic Growth.
  • Economic Development: It includes raising living standards and the quality of life, as seen in indicators like literacy rates, life expectancy, and access to healthcare.
    • Economic Development occurs when a country undergoes different economic developments that enhance quality of life, such as a decline in poverty and unemployment, a narrowing of wealth and income disparities, a rise in literacy rates, and improvements in health and sanitation.
    • An essential aspect of development is to enable the maximum number of people to experience the fruits of development.
    • This component of progress cannot be captured by ideas of per capita income (per capita GDP or per capita NSDP).
    • Non-financial social indicators with multiple dimensions are better indicators of a society’s overall economic progress.

 

Question 2

Consider the following statements:

  1. The quantity of imported edible oils is more than the domestic production of edible oils in the last five years.

  2. The Government does not impose any customs duty on all the imported edible oils as a special

Which of the statements given above is/are correct?

(a) 1 only

(b) 2 only

(c) Both 1 and 2

(d) Neither 1 nor 2

Ans: a

Sub-Theme: Agricultural commodity Trading

Statement 1 is correct: India occupies a prominent position in the world oilseeds industry with contribution of around 10% in worldwide production. But the demand of edible oils (extracted from oilseeds in addition to palm oil) is significantly higher than the domestic production, leading to dependence on imports (60% of requirement).

Statement 2 is incorrect: Government imposes customs duty on edible oils to safeguard the interests of the domestic oil crushing industry. The duty on two major edible oils, namely crude sunflower seed oil and crude canola/ rapeseed/mustard is 25% while crude soybean oil attracts 30% duty.

 

Question 3

Consider the following:

  1. Areca nut

  2. Barley

  3. Coffee

  4. Finger millet

  5. Groundnut

  6. Sesamum

  7. Turmeric

The Cabinet Committee on Economic Affairs has announced the Minimum Support Price for which of the above?

(a) 1, 2, 3 and 7 only

(b) 2, 4, 5 and 6 only

(c) 1, 3, 4, 5 and 6 only

(d) 1, 2, 3, 4, 5, 6 and 7

Ans: b

Sub-Theme: MSP

  • The Commission on Agricultural Costs and Prices’ (CACP) recommendations serve as the foundation for the Being a statutory agency, the CACP provides separate reports outlining its suggested pricing for the Kharif and Rabi seasons. Initially, MSP covered paddy, rice, wheat, jowar, bajra, maize, ragi (Finger Millet), barley, gram, tur, moong, urad, sugarcane, groundnut, soybean, sunflower seed, rapeseed and mustard, cotton, jute and tobacco.
  • Along with the edible oilseeds previously covered by the Commission, Niger seed and Sesame were included in the MSP Scheme of CACP as of 1994-1995. Similar to this, in 2001-2002, the government expanded the Commission’s mandate by adding lentil (masur).

 

Question 4

Consider the following statements:

1. Capital Adequacy Ratio (CAR) is the amount that banks have to maintain in the form of their own funds to offset any loss that banks incur if the account-holders fail to repay dues.

2. CAR is decided by each individual bank.

Which of the statements given above is/are correct?

(a) 1 only

(b) 2 only

(c) Both 1 and 2

(d) Neither 1 nor 2

Ans: a

Sub-Theme: Measures of NPA Resolution

Statement 1 is correct: Capital Adequacy Ratio (CAR) is the ratio of a bank’s capital in relation to its risk weighted assets and current liabilities.

Statement 2 is incorrect: CAR is decided by central banks and bank regulators to prevent commercial banks from taking excess leverage and becoming insolvent in the process.

 

Question 5

Which one of the following statements correctly describes the meaning of legal tender money?

(a) The money which is tendered in courts of law to defray the fee of legal cases

(b) The money which a creditor is under compulsion to accept in settlement of his claims

(c) The bank money in the form of cheques, drafts, bills of exchange etc.

(d) The metallic money in circulation in a country

Ans: b         

Sub-Theme: Money as a legal tender

Legal tender is any official medium of payment recognized by law which the creditor is obligated to accept towards repayment of a debt.

 

Question 6

Consider the following statements:

1. The Reserve Bank of India manages and services Government of India Securities but not any State Government Securities.

2. Treasury bills are issued by the Government of India and there are no treasury bills issued by the state Governments.

3. Treasury bills are issued at a discount from the par value.

Which of the statements given above is/are correct?

(a) 1 and 2 only

(b) 3 only

(c) 2 and 3 only

(d) 1, 2 and 3

Ans: c         

Sub-Theme: Debt Instrument

Statement 1 is incorrect: The RBI is the debt manager for both the Central Government and the State Governments. RBI manages the debt of state governments on the basis of separate agreements.

Statement 2 is correct: Treasury bills are money market instruments, are short term debt instruments issued by the Government of India and are presently issued in three tenors, namely, 91 days, 182 days and 364 days.

Statement 3 is correct: Treasury bills are issued at a discount from the par value (also known as the face value) of the bill, meaning the purchase price is less than the face value of the bill. For example, a $1,000 bill might cost the investor $950 to buy the product.

 

Question 7

With reference to the governance of public sector banking in India, consider the following statements:

1. Capital infusion into public sector banks by the Government of India has steadily increased in the last decade.

2. To put the public sector banks in order, the merger of associate banks with the parent State Bank of India has been affected.

Which of the statements given above is/are correct?

(a) 1 only

(b) 2 only

(c) Both 1 and 2

(d) Neither 1 nor 2

Ans: b          

Sub-Theme: Government policies

Statement 1 is incorrect: Capital infusion into public sector banks by the Government of India has not steadily increased in the last decade.

Statement 2 is correct: Merger of associate banks with the parent State Bank of India has been affected to create improved operational efficiency and economies of scale. It will also result in improved risk management and unified treasury operations.

 

Question 8

India enacted the Geographical Indications of Goods (Registration and Protection) Act, 1999 in order to comply with the obligations to:

(a) ILO

(b) IMF

(c) UNCTAD

(d) WTO

Ans: d

Sub-Theme: IP Related issues/WTO

  • Option (d) is correct: Geographical Indications (GI) are one of the eight intellectual property items coming under WTO’s TRIPS (Trade-Related Intellectual Property Rights).
  • According to the WTO, “Geographical indications are indications which identify a good as originating in the territory of a member, or a region or locality in that territory, where a given quality, reputation or other characteristics of the good is essentially attributable to its geographical origin.”
  • India enacted the Geographical Indication of Goods (Registration and Protection) Act, 1999 which came into force with effect from September 15, 2003, to comply with India’s obligations under the TRIPS agreement.
  • TRIPS Agreement: It is an international legal agreement between all the member nations of the World Trade Organization (WTO) to provide strong protection for Intellectual Property Rights. It came into effect on 1 January 1995.
  • The Controller General of Patents, Designs & TradeMarks (CGPDT), (under the Dept of Industrial Policy and Promotion of the Ministry of Commerce and Industry) is the ‘Registrar of Geographical Indications’. The CGPDT directs and supervises the functioning of the Geographical Indications Registry (GIR).

 

Question 9

Consider the following statements:

Human capital formation as a concept is better explained in terms of a process which enables:

1. Individuals of a country to accumulate more capital.

2. Increasing the knowledge, skill levels and capacities of the people of the country.

3. Accumulation of tangible wealth.

4. Accumulation of intangible wealth.

Which of the statements given above is/are correct?

(a) 1 and 2

(b) 2 only

(c) 2 and 4

(d) 1, 3 and 4

Ans: c

Sub-Theme: Human Capital

Statement 1 is incorrect: As per RBI, Gross capital formation refers to the aggregate of gross additions to fixed assets (that is fixed capital formation) plus change in stocks during the counting period.

Statement 2 is correct: OECD defines human capital as “the knowledge, skills, competencies and other attributes embodied in individuals or groups of individuals acquired during their life and used to produce goods, services or ideas in market circumstances”. Human capital formation is the outcome of investments in education, health, on-the job training, migration and information.

2 5

Statement 3 is incorrect: Tangible capital is also GCF as it involves largely infrastructural components.

Statement 4 is correct: Intangible wealth of a nation comprises the skilled population, human resource base, culture, arts etc.

 

Question 10

Despite being a high saving economy, capital formation may not result in significant increase in output due to:

(a) Weak administrative machinery

(b) Illiteracy

(c) High population density

(d) High capital-output ratio

Ans: d

Sub-Theme: Capital Formation

  • Capital formation entails building up a nation’s actual capital stock. In other words, capital creation entails creating new capital goods, such as factories, machinery, tools, materials, electricity,  transport equipment, etc., all of which are used to produce goods in the future.
  • Capital Output Ratio (COR): It serves as a measure for the proportional increase in capital formation needed to produce an increase in COR is a measure of capital required for producing one unit of output. If capital to output ratio is high, then capital formation may not result in a significant increase in the output.

 

Question 11

If a commodity is provided free to the public by the Government, then

(a) The opportunity cost is Zero.

(b) The opportunity cost is ignored.

(c) The opportunity costs are transferred from the consumers of the product to the tax-paying

(d) The opportunity cost is transferred from the consumers of the product to the Government.

Ans: c        

Sub-Theme: Opportunity Cost

Opportunity cost represents the benefits an individual, investor, or business misses out on when choosing one alternative over another. If a commodity is provided free to the public by the Government, then the opportunity cost is transferred from the consumers of the product to the tax-paying public. As per microeconomics, the opportunity cost is zero for free goods such as air and common goods such as fish/grazing land. For public goods such as street lights and defence, the opportunity cost is involved (The government could have spent that much money on street lights rather than on the military). So, the opportunity cost is not zero. Opportunity cost is also called Economic cost.

 

2017

 

Question 1

Which of the following has/have occurred in India after its liberalisation of economic policies in 1991?

  1. Share of agriculture in GDP increased enormously.

  2. Share of India’s exports in world trade increased

  3. FDI inflows increased.

  4. India’s foreign exchange reserves increased enormously.

Select the correct answer using the codes given below:

(a) 1 and 4 only

(b) 2, 3 and 4 only

(c) 2 and 3 only

(d) 1, 2, 3 and 4

Ans: b

Sub-Theme: Economic Reform

Statement 1 is incorrect: India’s agriculture sector has shown a gradual decline in contribution to the Indian economy post-reform. India’s traditional occupation, agriculture, currently makes up only 15% of the GDP, down from 29% in 1991.

Statement 2 is correct: India is seen as a successful exporter of IT software, auto parts, textiles, and engineering goods in the reform period. Thus, the share of India’s exports in world trade increased after its liberalisation of economic policies in 1991.

Statement 3 is correct: The Foreign investments including FDI and FII have increased from about US $ 100 million in 1990-91 to US $ 467 billion in 2012-13.

Statement 4 is correct: The opening up of the economy has led to rapid increase in foreign direct investment and foreign exchange reserves.

NOTE: This question came verbatim from Economics NCERT Class 11 Pg. 48. This highlights the importance of reading and understanding the concepts from NCERTs.

 

Question 2

What is/are the advantage/advantages of implementing the ‘National Agriculture Market’ scheme?

  1. It is a pan-India electronic trading portal for agricultural commodities.

  2. It provides the farmers access to nationwide markets, with prices commensurate with the quality of their produce.

Select the correct answer using the code given below:

(a) 1 only

(b) 2 only

(c) Both 1 and 2

(d) Neither 1 nor 2

Ans: c

Sub-Theme: Government Policies/e-technology to aid farmers.

Statement 1 is correct: National Agriculture Market (NAM) is a pan-India electronic trading portal which networks the existing APMC mandis to create a unified national market for agricultural commodities.

Statement 2 is correct: NAM promises more options for sale. It would increase farmers’ access to nationwide markets (with prices commensurate with the quality of their produce) through warehouse based sales and thus obviate the need to transport his produce to the mandi.

National Agriculture Market (NAM):

A national market for agricultural commodities is created by connecting the current APMC mandis through the pan-Indian electronic trading portal known as the National Agriculture Market (NAM).

Advantages of e-NAM:

For the Farmers: NAM promises more options for sale. It would increase farmers’ access to nationwide markets (with prices commensurate with the quality of their produce) through warehouse based sales and thus obviate the need to transport his produce to the mandi.

For the Local Trader: NAM provides access to a bigger national market for secondary trading in the mandi/market.

By participating directly in trading at the local mandi/market level through the NAM platform, bulk buyers, processors, exporters, etc. benefit from lower intermediation costs.

All of the major mandis in the States will gradually be incorporated into NAM, ensuring uniform processes for issuing licences, collecting fees, and moving products.

The NAM will also aid in the development of value chains for the nation’s most important agricultural commodities and support the movement and storage of agricultural products in a scientific manner.

 

Question 3

Which of the following statements is/are correct regarding the ‘Monetary Policy Committee (MPC)?

  1. It decides the RBI’s benchmark interest rates.

  2. It is a 12-member body including the Governor of RBI and is reconstituted every year.

  3. It functions under the chairmanship of the Union Finance Minister.

Select the correct answer using the code given below:

(a) 1 only

(b) 1 and 2 only

(c) 3 only

(d) 2 and 3 only

Ans: a          

Sub-Theme: Inflation Targeting/Banking and NBFCs

Statement 1 is correct: MPC determines the policy interest rate, which is required to achieve the inflation target.

Statement 2 is incorrect: As per the provisions of the RBI Act, Members of the Monetary Policy Committee is a six-member body of which three Members will be from the RBI and the other three Members of MPC will be appointed by the Central Government.

Statement 3 is incorrect: The Governor of the Reserve Bank of India is the ex-officio Chairperson of MPC.

 

Question 4

What is/are the most likely advantages of implementing ‘Goods and Services Tax’ (GST)?

1. It will replace multiple taxes collected by multiple authorities and will thus create a single market in India.

2. It will drastically reduce the ‘Current Account Deficit’ of India and will enable it to increase its foreign exchange reserves.

3. It will enormously increase the growth and size of the economy of India and will enable it to overtake China in the near future.

Select the correct answer using the code given below:

(a) 1 only

(b) 2 and 3 only

(c) 1 and 3 only

(d) 1, 2 and 3

Ans: a

Sub-Theme: Indirect Taxes/GST

Statement 1 is correct: GST is one indirect tax for the whole nation, which will make India one unified common market.

Statement 2 is incorrect: There is no information which says GST will drastically reduce the Current account deficit (CAD) of India.

Statement 3 is incorrect: GST implementation neither ensures enormous growth of the Indian economy nor does it enable India to take over China.

Goods and Services Tax (GST)

Definition It is a comprehensive tax levied on the manufacture, sale, and consumption of goods and services.
Established By the 101st Constitutional Amendment Act, on the lines of “One Nation One Tax”.
Powers The Parliament and the state legislatures have concurrent powers to implement GST
 

 

Features

•   Applicable on supply side

•   Destination based Taxation

•   Dual GST (Centre and the States simultaneously levying tax on a common base.)

•   GST rates to be mutually decided (CGST, SGST & IGST)

•   Multiple Rates

Merged Taxes Central Value Added Tax, Additional Customs Duty, Special Additional Duty of Customs, Central Sales Tax, Service Tax, State VAT (Sales tax)
Tax Slabs 5%, 12%, 18% and 28%.
 

3-Tiers of GST

1.       Centre levies the CENTRAL GST (CGST)

2.       State levies STATE GST (SGST)

3.       Centre levies INTEGRATED GST (IGST) on transactions

 

IGST

•   When the commodity is produced in one state and is traded to another state (interstate trade).

•   In this case, the share of SGST should go to the consuming state (as the GST is a destination- based tax).

 

 

Compensation

•   Parliament will compensate for any loss faced by the state

•   As per the GST Act, states are guaranteed compensation for any revenue shortfall below 14% growth (base year 2015-16) for the first five years ending 2022.

•   GST compensation is paid using funds specifically collected as compensation cess- is levied on products considered to be sin or luxury goods.

 

Input Tax Credit

•   It is a mechanism to avoid cascading of taxes.

•   It means at the time of paying tax on output, one can reduce the tax one has already paid on inputs and just pay the balance amount.

•   Cross utilisation of input tax credit is available

Reverse Charge Mechanism •   The receiver becomes liable to pay the tax, i.e., the chargeability gets reversed.

•   Self-invoicing is to be done when you have purchased from an unregistered supplier and such purchase of goods or services falls under reverse charge.

 

 

 

E-Way Bill System

It is an electronic waybill for movement of goods which can be generated on the e-Way Bill Portal

Objective:

•   Facilitate faster movement of goods.

•   Improve the turnaround time of vehicles.

•   It helps track intra-state as well as inter-state movements of goods of value exceeding Rs 50,000, for sales beyond 10 km in the Goods and Services Tax (GST) regime.

 

 

Commodities Outside Gst

•   Alcohol for human consumption,

•   Petroleum products,

•   Electricity, The supply of goods to the SEZ,

•   Supply of goods that come under zero rate,

•   Fresh vegetables, fresh milk, cereal, meat etc,

•   Raw materials.

NOTE: Lottery, Gambling and Betting are also Taxable Under the Goods and Services Tax (GST) Act, 2017

 

Advantages Disadvantages
Lesser compliance (returns, maintaining books of record, issuance of invoices). A limited territory of business.
Limited tax liability. No Input Tax Credit available to composition dealers
High liquidity as taxes are at a lower rate. The taxpayer will not be eligible to supply non- taxable goods under GST such as alcohol and goods through an e-commerce portal.

 

Question 5

Consider the following statements:

  1. Tax revenue as a percent of GDP of India has steadily increased in the last decade.

  2. Fiscal deficit as a percent of GDP of India has steadily increased in the last decade.

Which of the statements given above is/are correct?

(a) 1 only

(b) 2 only

(c) Both 1 and 2

(d) Neither 1 nor 2

Ans: d

Sub-Theme: Tax Revenue/Fiscal Deficit

Statement 1 is incorrect: Service tax, personal income tax and corporation tax have been reduced in the recent past to boost aggregate demand, so as to protect the economy from global recession. Excise duty rates have recently been lowered in order to raise aggregate demand and safeguard the economy from a worldwide recession; as a result, tax revenue as a percentage of India’s GDP hasn’t increased over the past ten years.

Statement 2 is incorrect: Fiscal deficit as a percent of GDP of India has not steadily increased in the last decade.

 

Question 6

With reference to the ‘National Intellectual Property Rights Policy’, consider the following statements:

1. It reiterates India’s commitment to the Doha Development Agenda and the TRIPS

2. Department of Industrial Policy and Promotion is the nodal agency for regulating intellectual property rights in India.

Which of the above statements is/are correct?

(a) 1 only

(b) 2 only

(c) Both 1 and 2

(d) Neither 1 nor 2

Ans: c

Sub-Theme: IP Related issues

Statement 1 is correct: National Intellectual Property Rights Policy was introduced as India’s commitment to the Doha Development Agenda (trade negotiation round of the WTO) and the TRIPS Agreement. It is an international legal agreement between all the member nations of the World Trade Organization (WTO) to provide strong protection for Intellectual Property Rights. It came into effect on 1 January 1995.

Statement 2 is correct: The Department of Industrial Policy and Promotion (DIPP) has been nominated as the nodal department to coordinate, guide and oversee implementation and future development of IPRs in India.

National Intellectual Property Rights Policy:

  • In terms of IP awareness and development, commercialization, and enforcement, it seeks to create an ecosystem in the nation that is supportive of innovation and creativity.
  • The nodal department has been nominated as being the Department of Industrial Policy and Promotion (DIPP).
  • This policy was introduced as India’s commitment to the Doha Development Agenda and the TRIPS Agreement.
  • National Intellectual Property Rights Policy encompasses and brings to a single platform all IPRs. It aims to incorporate and adapt global best practices to the Indian

TRIPS Agreement:

  • It’s an agreement on Trade-Related Aspects of Intellectual Property Rights (TRIPS). It is an international legal agreement between all the member nations of the World Trade Organization (WTO). It came into effect on 1 January 1995.

 

2016

 

Question 1

Which of the following is/are the advantage/ advantages of practicing drip irrigation?

  1. Reduction in weed

  2. Reduction in soil salinity

  3. Reduction in-soil erosion

Select the correct answer using the code given below.

(a) 1 and 2 only

(b) 3 only

(c) 1 and 3 only

(d) None of the above is an advantage of practising drip irrigation

Ans: c

Sub-Theme: Sustainable Agriculture

Drip Irrigation:

  • It is a form of irrigation where water is applied near the plant root through emitters or drippers, on or below the soil surface, at a low rate varying from 2-20 litres per Frequent watering helps to maintain the optimum level of soil moisture.
  • The most effective irrigation technique is drip irrigation, which may be used for a wide range of crops, including vegetables, orchard crops, flowers, and plantation

Advantages of Drip Irrigation:

  • Maximum use of available
  • No water being available to
  • Maximum crop
  • High efficiency in the use of
  • Weed growth is reduced and restricts the population of potential hosts.
  • Low labour and relatively low operation
  • No soil
  • Improved infiltration in soil of low
  • Ready adjustment to sophisticated automatic control.
  • No runoff of fertilizers into
  • Less water is lost to evaporation than during surface irrigation.
  • Improves seed
  • Decreased tillage

Disadvantages of Drip Irrigation:

  • Sensitivity to clogging
  • Moisture distribution problem
  • Salinity hazards
  • High cost compared to
  • For design, installation, and operation, high competence is needed.

 

Question 2

Which one of the following is the purpose of ‘UDAY’, a scheme of the Government?

(a) Providing technical and financial assistance to startup entrepreneurs in the field of renewable sources of energy

(b) Providing electricity to every household in the countries by 2018

(c) Replacing the coal-based power plants with natural gas, nuclear, solar, wind and tidal power plants over a period of time

(d) Providing for financial turnaround and revival of power distribution companies

Ans: d

Sub-Theme: Government Policies

UDAY Scheme

  • It is the Government of India’s financial recovery and revival package for India’s electricity distribution firms (DISCOMs), which aims to permanently fix the financial crisis of the power distribution industry.
  • Objectives:
    • Improving operational efficiencies of discoms,
    • Reduction of cost of power,
    • Reduction in interest cost of discoms and
    • Enforcing financial discipline on discoms through alignment with state finances.

 

Question 3

What is/are the purpose/purposes of ‘District Mineral Foundations’ in India?

1. Promoting mineral exploration activities in mineral-rich districts

2. Protecting the interests of the persons affected by mining operations

3. Authorizing State Governments to issue licenses for mineral exploration

Select the correct answer using the code given below.

(a) 1 and 2 only

(b) 2 only

(c) 1 and 3 only

(d) 1, 2 and 3

Ans: b

Sub-Theme: Government policies

Statement 1 is incorrect: DMFs don’t work in promotion of mineral exploration activities in mineral rich districts, rather it work on the benefit of the persons and areas affected mining related operations.

Statement 2 is correct: The main objective of DMFs is to work for the interest of the benefit of the persons and areas affected mining related operations in such manner as may be prescribed by the State Government.

Statement 3 is incorrect: It does not authorise states to issue a licence for mineral exploration. Additional Information:

District Mineral Foundations (DMFs):

  • DMFs were instituted under the Mines and Minerals (Development and Regulation) (MMDR) Amendment Act 2015.
  • They are non-profit trusts created to operate in the interest of those impacted by mining activities and for their own
  • Miners’ contributions provide the funding for it.
  • Objective: To work for the interest of the benefit of the persons and areas affected mining related operations in such manner as may be prescribed by the State
  • Jurisdiction: Its manner of operation comes under the jurisdiction of the relevant State government.

NOTE: Through our general understanding we know that DMFs work for the interest and benefit of persons and areas affected by mining related operations, hence, with this simple understanding we can rule out statement 3 easily and by eliminating this single statement we can easily arrive at the correct answer.

 

Question 4

With reference to ‘Bitcoins’, sometimes seen in the news, which of the following statements is/are correct?

1. Bitcoins are tracked by the Central Banks of the countries.

2. Anyone with a Bitcoin address can send and receive Bitcoins from anyone else with a Bitcoin address.

3. Online payments can be sent without either side knowing the identity of the other.

Select the correct answer using the code given below.

(a) 1 and 2 only

(b) 2 and 3 only

(c) 3 only

(d) 1, 2 and 3

Ans: b

Sub-Theme: Cryptocurrency

Statement 1 is incorrect: Bitcoin is a digital currency that is not tied to a bank or government and allows users to spend money anonymously.

Statements 2 and 3 are correct: Bitcoins have become popular because transactions can be made anonymously, making the currency popular with libertarians as well as tech enthusiasts, speculators and criminals. Also, anyone with a Bitcoin address can send and receive Bitcoins from anyone else with a Bitcoin address.

Bitcoin:

  • Bitcoin is a digital currency that is not tied to any formal institution like bank or government and it also allows users to transact anonymously.
  • Generally the coins are created by users who “mine’’ them by lending computing power to verifying other users’ transactions.
  • They receive bitcoins in exchange. The buying and selling of coins also can be done on exchanges with U.S. dollars and other
  • Bitcoins became popular due to its anonymous transactions, making the currency popular with libertarians as well as tech enthusiasts, speculators and
  • Anyone with a Bitcoin address can send and receive Bitcoins from anyone else with a Bitcoin address.

 

Question 5

What is/are the purpose/purposes of the Government’s ‘Sovereign Gold Bond Scheme’ and ‘Gold Monetization Scheme’?

1. To bring the idle gold lying with Indian households into the economy.

2. To promote FDI in the gold and jewellery sector

3. To reduce India’s dependence on gold imports.

Select the correct answer using the code given below.

(a) 1 only

(b) 2 and 3 only

(c) 1 and 3 only

(d) 1, 2 and 3

Ans: c

Sub-Theme: Government policies

Statements 1 and 3 are correct: The primary objectives of these schemes were to reduce India’s gold imports and bring all the gold lying idle with individuals and households.

Statement 2 is incorrect: All Scheduled Commercial Banks excluding RRBs are eligible to implement the Scheme. Both the schemes are not intended to promote FDI in the gold and jewellery sector.

Gold Monetization Scheme and Sovereign Gold Bond Scheme:

  • The government had introduced three ambitious plans to lower the physical demand for gold and remove 20,000 tonnes of the precious metal from households where it was sitting inert and worth $800 billion.
  • PM launched the maiden sovereign gold bond, gold monetisation and the Indian gold coin scheme.
  • Objective: To reduce India’s gold imports and bring all the gold lying idle with individuals and Sovereign Gold Bond Scheme was launched under the Gold Monetisation Scheme in November 2015.
  • Sovereign Gold Bonds (SGBs) are government securities denominated in grams of gold issued by the RBI on behalf of the Government.
  • The Gold Monetization Scheme aims to minimise the nation’s dependency on gold imports by enabling the use of gold held by households and facilitating their mobilisation for productive uses.
  • Except for RRBs, all Scheduled Commercial Banks may implement the Scheme. Both programmes are not meant to encourage FDI in the gold and jewellery industry.

 

Question 6

The establishment of ‘Payment Banks’ is being allowed in India to promote financial Which of the following statements is/are correct in this context?

1. Mobile telephone companies and supermarket chains that are owned and controlled by residents are eligible to be promoters of Payment Banks.

2. Payment Banks can issue both credit cards and debit cards.

3. Payment Banks cannot undertake lending activities

Select the correct answer using the code given below.

(a) 1 and 2 only

(b) 1 and 3 only

(c) 2 only

(d) 1, 2 and 3

Ans: b        

Sub-Theme: Banking Licences

Statement 1 is correct: Eligible promoters for payment banks include Mobile phone companies, supermarket chains, public sector entities, real sector cooperatives, etc.

Statement 2 is incorrect: Credit risk is not involved with the Payments Bank. It can carry out most banking operations but cannot advance loans or issue credit cards.

Statement 3 is correct: As per RBI guidelines, Payment Banks are not permitted to engage in lending activities.

 

Question 7

The term ‘Core Banking Solution’ is sometimes seen in the news. Which of the following statements best describes/describe this term?

1. It is a network of a bank’s branches which enables customers to operate their accounts from any branch of the bank on its network regardless of where they open their accounts.

2. It is an effort to increase RBI’s control over commercial banks through computerization.

3. It is a detailed procedure by which a bank with huge non-performing assets is taken over by another bank.

Select the correct answer using the code given below.

(a) 1 only

(b) 2 and 3 only

(c) 1 and 3 only

(d) 1, 2 and 3

Ans: a

Sub-Theme: Banking Functions: Core banking solution (CBS)

Statement 1 is correct: Core Banking Solution (CBS) is a network of bank branches, which allows customers to manage their accounts, and use various banking facilities from any part of the world.

Statement 2 is incorrect: RBI doesn’t use CBS as a tool to exercise control over commercial banks through computerization.

Statement 3 is incorrect: CBS has nothing to do with NPAs.

 

Question 8

What is/are the purpose/purposes of the ‘Marginal Cost of Funds based Lending Rate (MCLR)’ announced by RBI?

1. These guidelines help improve the transparency in the methodology followed by banks for determining the interest rates on advances.

2. These guidelines help ensure availability of bank credit at interest rates which are fair to the borrowers as well as the banks.

Select the correct answer using the code given below.

(a) 1 only

(b) 2 only

(c) Both 1 and 2

(d) Neither 1 nor 2

Ans: c

Sub-Theme: Banking Mechanisms/Functions

Objectives of MCLR:

  • Improving the transmission of the policy rate into lending rates of the bank
  • Bringing transparency in the methods followed by various banks for the determination of interest rate.
  • Ensuring the availability of bank loans at rates that are fair to both lenders and borrowers.

Enabling the lender and bank to be competitive and improve their worth in the long run.

 

Question 9

There has been a persistent deficit budget year after year. Which action/actions of the following can be taken by the Government to reduce the deficit?

1. Reducing revenue expenditure

2. Introducing new welfare schemes

3. Rationalizing subsidies

4. Reducing import duty

Select the correct answer using the code given below.

(a) 1 only

(b) 2 and 3 only

(c) 1 and 3 only

(d) 1, 2, 3 and 4

Ans: c

Sub-Theme: Budget

Statement 1 is correct: Reducing the revenue expenditure will certainly help in bridging the gap.

Statement 2 is incorrect: Introducing new welfare schemes will most likely result in increasing expenditure and so will expanding industries which would require capital infusion. These steps will increase the budget deficit.

Statement 3 is correct: Subsidies are a part of expenditure and rationalizing them would reduce the deficit.

Statement 4 is incorrect: Reducing import duty will result in further widening of the gap.


 

Question 10

Which of the following is/are included in the capital budget of the Government of India?

1. Expenditure on acquisition of assets like roads, buildings, machinery, etc.

2. Loans received from foreign governments

3. Loans and advances granted to the States and Union Territories

Select the correct answer using the code given below.

(a) 1 only

(b) 2 and 3 only

(c) 1 and 3 only

(d) 1, 2 and 3

Ans: d

Sub-Theme: Budget

Statement 1 is correct: Capital Expenditure includes the expenditure on the acquisition of land, building, machinery, equipment, creating assets such as roads and hospitals, repayment of government borrowings.

Statement 2 is correct: Capital receipts components are: Loan borrowings, disinvestments, funds received from the issue of shares or debentures, etc.

Statement 3 is correct: Loans, and advances by the central government to state and union territory governments, etc. are also included in the capital budget of the Government of India.

 

Question 11

With reference to ‘Financial Stability and Development Council’, consider the following statements:

1. It is an organ of NITI Aayog.

2. It is headed by the Union Finance Minister.

3. It monitors macroprudential supervision of the economy.

Which of the statements given above is/are correct?

(a) 1 and 2 only

(b) 3 only

(c) 2 and 3 only

(d) 1, 2 and 3

Ans: c

Sub-Theme: Banking and NBFCs

Statement 1 is incorrect: The Financial Stability and Development Council (FSDC) under the chairmanship of the Union Finance Minister was set up by the government as the apex-level forum in December 2010. It was set up much before NITI Aayog was even set up. Statement 2 is correct: The Chairman of the council is the finance minister.

Statement 3 is correct: The Council monitors macro-prudential supervision of the economy, which includes functioning of large financial conglomerates.

Financial Stability and Development Council (FSDC):

  • In order to serve as the apex level forum, the Financial Stability and Development Council (FSDC) was established by the government in December 2010.
  • The Chairman of the Council is the Union Finance Minister and its members include:
    • The heads of financial sector Regulators (RBI, SEBI, PFRDA, IRDA & FMC)
    • Finance Secretary and/or Secretary, Department of Economic Affairs,
    • Secretary,     Department    of     Financial Services,
    • Chief Economic Adviser.
  • The Council can invite experts to its meeting if required.
  • Functions:
    • To improve inter-regulatory coordination, institutionalize financial sector development, and strengthen the system for preserving financial stability.
    • To monitor macro-prudential super- vision of the economy. It assesses the functioning of the large financial conglomerates.

 

Question 12

Which of the following best describes the term ‘import cover’, sometimes seen in the news?

(a) It is the ratio of value of imports to the Gross Domestic Product of a country

(b) It is the total value of imports of a country in a year

(c) It is the ratio between the value of exports and that of imports between two countries

(d) It is the number of months of imports that could be paid for by a country’s international reserves

Ans: d         

Sub-Theme: BoP

Import cover serves as a crucial currency stability indicator. It measures the number of months of money available in the national bank to cover the cost of imports or the stock of foreign exchange reserves in terms of months of retained imports of goods as at end of year. Hence, it is the number of months of imports that could be paid for by a country’s international reserves.

 

Question 13

In the context of which of the following do you sometimes find the terms ‘amber box, blue box and green box’ in the news?

(a) WTO affairs

(b) SAARC affairs

(c) UNFCCC affairs

(d) India-EU negotiations on FTA

Ans: a

Sub-Theme: WTO

  • In WTO jargon Domestic agricultural subsidies are denoted by “Boxes,” specifically Green Box, Amber Box, and Blue Box.
    • Amber Box: It refers to a category of domestic support or subsidies under the WTO’s Agreement on Agriculture. With limited exceptions, all domestic assistance policies that distort commerce and production are classified under amber box.
    • Blue Box: This is the “amber box with conditions” – conditions designed to reduce distortion. Any support that would normally be in the amber box, is placed in the blue box if the support also requires farmers to limit production by imposing production quotas. It is an exemption from the general rule that all subsidies linked to production must be reduced or kept within defined minimal levels.
    • Green Box: Subsidies that do not distort trade or at most cause minimal distortions are placed in Green Box. These subsidies have to be government- funded (not by charging consumers higher prices) and must not involve price support.

 

Question 14

With reference to the International Monetary and Financial Committee (IMFC), consider the following statements:

1. IMFC discusses matters of concern affecting the global economy, and advises the International Monetary Fund (IMF) on the direction of its

2. The World Bank participates as an observer in IMFC’s meetings.

Which of the statements given above is/are correct?

(a) 1 only

(b) 2 only

(c) Both 1 and 2

(d) Neither 1 nor 2

Ans: c

Sub-Theme: Various Financial Committee/ Organisations

Statement 1 is correct: International Monetary and Financial Committee (IMFC) advises and reports to the IMF Board of Governors on the supervision and management of the international monetary and financial system. Although it lacks formal decision- making authority, in practice it has developed into a key tool for providing strategic direction to the work and policies of the IMF.

Statement 2 is correct: A number of international institutions, including the World Bank, participate as observers in the IMFC’s meetings.

 

Question 15

India’s ranking in the ‘Ease of Doing Business Index’ is sometimes seen in the Which of the following has declared that ranking?

(a) Organization for Economic Cooperation and Development (OECD)

(b) World Economic Forum

(c) World Bank

(d) World Trade Organization (WTO)

Ans: c

Sub-Theme: Reports and Indexes/ World Bank Reports

  • Ease of Doing Business Index is published by the World Bank. It is an aggregate figure that includes different parameters which define the ease of doing business in a It is calculated by adding up the distance to frontier ratings of various economies. The “regulatory best practices” for conducting business serve as the basis for the distance to frontier score, which compares economies based on that parameter. The index for ease of doing business is calculated based on the following indicators: Construction permits, registration, obtaining financing, tax payment mechanisms, etc.
  • Other Reports of World Bank:
    • Remittance Report
    • Universal Health Coverage Index
    • The Service trade restriction index
    • Ease of Living Index
    • Global Economic Prospect (GEP) Report
    • World Development Report
    • Human Capital Index

 

2015

 

Question 1

The Fair and Remunerative Price (FRP) of sugarcane is approved by the:

(a) Cabinet Committee on Economic Affairs.

(b) Commission for Agricultural Costs and Prices.

(c) Directorate of Marketing and Inspection, Ministry of Agriculture.

(d) Agricultural Produce Market committee.

Ans: a

Sub-Theme: Government Policies

Fair and Remunerative Prices (FRP):

  • The Sugarcane Control Order, 1966, published under the Essential Commodities Act (ECA), 1955, specifies that FRP must be paid nationwide within 14 days of the cane’s delivery date.
  • The Cabinet Committee on Economic Affairs (CCEA) announced the decision following the Commission on Agricultural Costs and Prices’ (CACP) recommendation.
  • The report by the Rangarajan Committee on restructuring the sugarcane sector serves as the foundation for the FRP.

 

Question 2

In India, markets in agricultural products are regulated under the

(a) Essential Commodities Act, 1955

(b) Agricultural Produce Market Committee Act enacted by States

(c) Agricultural Produce (Grading and Marking) Act, 1937

(d) Food Products Order, 1956 and Meat and Food Products Order, 1973

Ans: b

Sub-Theme: Government Policies

Agricultural Produce Market Committee (APMC):

  • Agricultural Produce Market Committee (APMC) under the State Government regulates the notified agricultural produce and livestock.
  • The State Government’s Agricultural Produce Market Committee (APMC) oversees the listed agricultural products and livestock.
  • According to Schedule 7 of the Indian Constitution, agriculture is a state subject.
  • The entire state’s area has been divided up and designated as a market area (Yard Mandis), and market management is carried out by Market Committees established by the state governments.
  • No person or organisation is permitted to engage in wholesale marketing operations after a certain area is designated as a market area and comes under the control of a Market In order to transact, buyers must also get unique licences from each APMC.
  • It is a component of government policy aimed at ensuring food security, paying remunerative prices to the farmers, and providing fair prices to consumers.

 

Question 3

Consider the following statements:

1. The Accelerated Irrigation Benefits Programme was launched during 1996-97 to provide loan assistance to poor farmers.

2. The Command Area Development Programme was launched in 1974-75 for the development of water-use efficiency.

Which of the statements given above is/are correct?

(a) 1 only

(b) 2 only

(c) Both 1 and 2

(d) Neither 1 nor 2

Ans: b

Sub-Theme: Sustainable Agriculture

Statement 1  is   incorrect:   Accelerated Irrigation Benefits Programme was launched in 1996-97 to give loan assistance to the states to help them to develop their major irrigation projects which are in an advanced stage of completion. Its objective was to expedite completion of ongoing irrigation projects.

Statement 2 is correct: The command area development programme was launched in 1974-75 to bridge the gap between irrigation potential and actual utilized major and medium irrigation schemes. Delivery of irrigation water to the fields is the ultimate objective of this scheme. The plan includes the creation of field channels, land contouring, and rotational water supply to ensure equitable distribution. It was restructured into the Command Area Development and Water Management Program 2004.

 

Question 4

The substitution of steel for wooden ploughs in agricultural production is an example of

(a) Labour-augmenting technological progress

(b) Capital-augmenting technological progress

(c) Capital-reducing technological progress

(d) None of the above

Ans: b

Sub-Theme: Agricultural Infrastructure

  • Labour Augmenting is technology that increases skills and productivity of the existing labour force (example – teaching people how to use the computer).
  • Capital Augmenting technology enhances the productivity of existing capital goods. In this case, replacement of wood by steel, increases the productivity of plough.

 

Question 5

Which one of the following best describes the main objective of ‘Seed Village Concept’?

(a) Encouraging the farmers to use their own farm seeds and discouraging them to buy the seeds from others

(b) Involving the farmers for training in quality seed production and thereby to make available quality seeds to others at appropriate time and affordable cost

(c) Earmarking some villages exclusively for the production of certified seeds

(d) Identifying the entrepreneurs in villages and providing them technology and finance to set up seed companies

Ans: b

Sub-Theme: Government Policies

Seed village Concept:

  • A village is referred to as a “seed village” if a trained group of farmers produces seeds for a variety of crops and provides for the requirements of themselves, their fellow villagers, and villagers in nearby villages at an affordable price.
  • Major focus areas are:
    • Clustering  or concentrating seed production in a small area
    • Introducing new, high-yielding types to replace the current local ones.
    • Increasing the seed production
    • To meet the local demand, timely supply and reasonable cost
    • Self-sufficiency and self-reliance of the village
    • Increasing the seed replacement rate
    • This allows seed to be available at the door steps of farms at an appropriate time and at an affordable cost. It also facilitates fast spread of new cultivars of different kinds.

 

Question 6

With reference to inflation in India, which of the following statements is correct?

(a) Controlling the inflation in India is the responsibility of the Government of India only

(b) The Reserve Bank of India has no role in controlling the inflation

(c) Decreased money circulation helps in controlling the inflation

(d) Increased money circulation helps in controlling the inflation

Ans: c

Sub-Theme: Inflation Measures to Combat Inflation

Inflation is the rise in prices of goods and services within a particular economy wherein, the purchasing power of consumers decreases, and the value of the cash holdings erode.

Additional Information:

  • Reserve Bank of India is the authority to control inflation through monetary policies which it does by increasing bank rates, repo rates, cash reserve ratio, buying dollars, regulating money supply and availability of
  • Controlling Inflation: Decreased money circulation helps in controlling the Inflation.
  • Causes that Lead to Inflation are: Increase in demand, reduction in supply, demand- supply gap, excess circulation of money, increase in input costs, devaluation of currency, rise in wages among others.
Monetary Policy Measures Fiscal Policy Measures Other Measures
•   Increase of the Bank rate. •   Reduces the private spending by increasing taxes •   Price control by government as a short -term measure
•   Make      borrowing       costly by increasing interest rates. •   Reduces the government spending •   Import controls imposed by the government.
•   Increasing tendency to save •   Bringing more people under tax coverage •   Restricting the wage increase by companies
•   Controlling the credit-creation •   Introducing new taxes and cess
•   Conducting open market operations

•   Increasing the Repo Rate, Bank Rate, CRR, SLR and other policy rates.

 

NOTE: This is absolutely a straightforward and easy question. Anyone with a thorough reading of NCERTs and Ramesh Singh could easily answer this question.

 

Question 7

‘Pradhan Mantri Jan-Dhan Yojana’ has been launched for:

(a) Providing housing loans to poor people at cheaper interest rates.

(b) Promoting women’s Self-help Groups in backward areas.

(c) Promoting financial inclusion in the country.

(d) Providing financial help to the marginalized communities.

Ans: c

Sub-Theme: Government Policies

Financial Inclusion & Access to financial products & services are one of the key objectives of PMJDY Scheme.

 

Question 8

When the Reserve Bank of India reduces the Statutory Liquidity Ratio by 50 basis points, which of the following is likely to happen?

(a) India’s GDP growth rate increases drastically

(b) Foreign Institutional Investors may bring more capital into our country

(c) Scheduled Commercial Banks may cut their lending rates

(d) It may drastically reduce the liquidity to the banking system

Ans: c

Sub-Theme: Monetary Policy Tools

The Statutory Liquidity Ratio (SLR) cut by the RBI is likely to give more room for banks to cut rates. Scheduled Commercial Banks may cut their lending rates.

The portion of deposits that banks must hold in the form of gold or government securities before extending credit to consumers is known as the SLR.

 

Question 9

‘Basel III Accord’ or simply ‘Basel III’ often seen in the new, seeks to:

(a) Develop national strategies for the conservation and sustainable use of biological diversity

(b) Improve banking sector’s ability to deal with financial and economic stress and improve risk management

(c) Reduce the greenhouse gas emissions but places a heavier burden on developed countries

(d) Transfer technology from developed countries to poor countries to enable them to replace the use of chlorofluorocarbons in refrigeration with harmless chemicals

Ans: b

Sub-Theme: BASEL III Norms

BASEL-III NORMS

  • The Basel Accords are 3 series of banking regulations (Basel I, II, and III) set by the Basel Committee on Bank Supervision (BCBS).
  • Basel Accords  aim  to  strengthen the regulation, supervision and risk management of banks.
  • Presently the Indian banking system follows Basel II norms.
  • Under Basel III, a bank’s tier-1 and tier-2 assets must be at least 10.5% of its risk – weighted assets.
 

 

Tier-1

• It is the primary funding source of the bank.

• Tier-1 capital consists of shareholders’ equity and retained earnings.

 

 

 

 

 

Tier-2

• Tier-2 capital includes revaluation reserves, hybrid capital instruments and subordinated term debt, general loan – loss reserves, and undisclosed reserves.

• Tier-2 capital is considered less reliable than Tier 1 capital because it is more difficult to accurately calculate and more difficult to liquidate

 

Question 10

With reference to Indian economy, consider the following:

  1. Bank rate

  2. Open market operations

  3. Public debt

  4. Public Revenue

Which of the above is/are component/components of Monetary Policy?

(a) 1 only

(b) 2, 3 and 4

(c) 1 and 2

(d) 1, 3 and 4

Ans: c

Sub-Theme: RBI Monetary Policy

The monetary policy tool is implemented by the RBI through open market operations, bank rate, CRR, SLR, the repo rate, reverse repo rate. Statement 1 is correct: The bank rate is the rate of interest at which RBI charges while lending loans to a commercial bank.

Statement 2 is correct: The Open market operations are conducted by the RBI by way of sale and purchase of Government Securities (G-Secs) to adjust liquidity in the market.

Statements 3 and 4 are incorrect: Public Revenue and debt are not part of monetary policy. These are related to fiscal policy.

 

Question 11

With reference to the Fourteenth Finance Commission, which of the following statements is/are correct?

1. It has increased the share of States in the central divisible pool from 32 percent to 42 percent.

2. It has made recommendations concerning sector- specific grants.

Select the correct answer using the code given below.

(a) 1 only

(b) 2 only

(c) Both 1 and 2

(d) Neither 1 nor 2

Ans: a

Sub-Theme: Finance Commission

Statement 1 is correct: The Fourteenth Finance Commission has radically enhanced the share of the states in the central divisible pool from the current 32% to 42% which is the biggest ever increase in vertical tax devolution.

Statement 2 is incorrect: The Fourteenth Finance Commission has not made any recommendation concerning sector specific grants unlike the Thirteenth Finance Commission.

Finance Commission

Constitutional Provision Article 280, Part-XII
Appointment By PRESIDENTIAL order.
 

Composition

Chairman + 4 other members.
 

Qualification

Determined by PARLIAMENT
 

Tenure

Constituted by the president of India every fifth year or at such earlier time as he considers necessary.
 

 

Other Facts

•   Recommendations are advisory in nature.

•   Balancing wheel of fiscal federalism.

 

15th Finance Commission
Chairman •   N.K. Singh
 

Key Recommendations

 

Vertical Devolution:

  • The share of states in the centre’s taxes is recom-mended to be decreased to 41%.
  • 1%  less   than   14th commission  (42%), that 1% is to provide for the newly formed Union Territories of J&K and  Ladakh  from  the resources of the central government.

Horizontal Devolution:

  • The  criteria  and  the weights assigned   for horizontal    devolution are:
    • Population – 15%
    • Area – 15%
    • Forest & Ecology – 10%
    • Income Distance – 45%
    • Tax and Fiscal Efforts – 2.5%
    • Demographic
    • Performance – 12.5%
•   Recommendation  to create  a  non-lapsable pool  for  the  defence and  internal security sector under the Public Accounts of India.

 

Question 12

With reference to the Union Government, consider the following statements:

1. The Department of Revenue is responsible for the preparation of the Union Budget that is presented to the Parliament.

2. No amount can be withdrawn from the Consolidated Fund of India without the authorization from the Parliament of India.

3. All the disbursements made from Public Account also need the authorization from the Parliament of India.

Which of the statements given above is/are correct?

(a) 1 and 2 only

(b) 2 and 3 only

(c) 2 only

(d) 1, 2 and 3

Ans: c

Sub-Theme: Public Finance

Statement 1 is incorrect: The budget is prepared by the Department of Economic Affairs.

Statement 2 is correct: Consolidated Fund of India (Article 266) is a fund to which all receipts are credited and all payments are debited. No money out of this fund can be appropriated (issued or drawn) except in accordance with parliamentary law.

Statement 3 is incorrect: Provident fund deposits, savings bank deposits, remittances etc are credited to the Public Account. The Public Account is operated by the executive action i.e. payment from this account can be made without Parliamentary appropriation.

Annual Financial Statement (Budget): Article 112

  • The term ‘Budget’ is nowhere mentioned in the Constitution.
  • Budget is referred to as the Annual Financial Statement in the constitution under Art. 112.
  • The Budget is a statement of the Government estimated receipts and expenditure in a financial year starting from APRIL 1 and ending on 31 MARCH.
  • Contains Appropriation Bill (Expenditure Side) and Finance Bill (Receipts).
  • Revenue Account: Receipts and expenditure that relate to the current financial year only (revenue budget)
  • Capital Account: Concern with the assets and liabilities of the government (capital budget).
  • On the Acworth Committee’s recommenda-tion, the Rail Budget was separated from the General Budget in However, it was merged again in 2017.
  • Some recent reforms related to budget:
    • Budget is preponed to 1st FEB
    • Railway budget merged with general budget
    • No vote on account
    • Plan and non-plan expenditure removed.
  • Presented in terms of:
    • Consolidated Fund of India,
    • Contingency Fund of India
    • Public Account of India

 

Question 13

A decrease in tax to GDP ratio of a country indicates which of the following?

Slowing economic growth rate Less equitable distribution of national income

Select the correct answer using the codes given

(a) 1 only

(b) 2 only

(c) Both 1 and 2

(d) Neither 1 nor 2

Ans: a

Sub-Theme: Economic Growth

Statement 1 is correct: A lower tax-to-GDP ratio puts pressure on the government to meet its fiscal deficit Targets.

Statement 2 is incorrect: Tax-GDP ratio does not indicate the distribution of national income in the economy.

Tax to GDP Ratio:

  • A tax-to-GDP ratio is a gauge of a nation’s tax revenue relative to the size of its economy as measured by gross domestic product (GDP).
  • The tax-to-GDP ratio is a measure of a nation’s tax revenue relative to the size of its economy. It determines how well a nation’s government uses its economic resources via
  • The tax to GDP ration is often higher in developed countries than in developing countries. A low tax to GDP ratio indicates sluggish economic The ratio represents that the government can finance its expenditure.
  • An economy’s tax buoyancy is robust if the tax to GDP ratio is higher. With a lower tax-to-GDP ratio, the government is under more pressure to stick to its budget deficit

 

Question 14

With reference to Indian economy, consider the following statements:

1. The rate of growth of Real Gross Domestic Product has steadily increased in the last decade.

2. The Gross Domestic Product at market prices (in rupees) has steadily increased in the last decade.

Which of the statements given above is/are correct?

(a) 1 only

(b) 2 only

(c) Both 1 and 2

(d) Neither 1 nor 2

Ans: b

Sub-Theme: Economic Growth

Statement 1 is incorrect: Real Gross Domestic Product is an inflation-adjusted measure that reflects the value of all goods and services produced by an economy in a given year (expressed in base-year prices). Real GDP makes comparing GDP from year to year and from different years more meaningful because it shows comparisons for both the quantity and value of goods and services. Rate of growth of Real Gross Domestic Product has fluctuated a lot during the last decade and decreased significantly in 2008-09 due to the global financial crisis.

Statement 2 is correct: Gross domestic product at market prices is the sum of added values of all activities which produce goods and services, plus taxes and minus subsidies on products. Its objective is to determine the total wealth generated by both private and public agents in a nation’s territory over a specific time frame. The most important component of national accounts, it shows the outcome of productive activity carried out by local producing units. Gross Domestic Product at market prices (in rupees) has increased and did not show a declining trend even once during the last decade as clearly seen from the planning commission data.

NOTE: India’s Gross Domestic Product (GDP) contracted by 7.3% in 2020-21. However, between the early 1990s until the pandemic hit the country, India grew at an average of around 7% every year. The question was asked in 2015 and hence, the explanation is provided accordingly.

 

Question 15

There has been a persistent deficit budget year after year. Which of the following actions can be taken by the government to reduce the deficit?

  1. Reducing revenue expenditure

  2. Introducing new welfare schemes

  3. Rationalising subsidies

  4. Expanding industries

Select the correct answer using the code given below.

(a) 1 and 3 only

(b) 2 and 3 only

(c) 1 only

(d) 1, 2, 3 and 4

Ans: a

Sub-Theme: Public Finance

Statement 1 is correct: Budget deficit is used to define a status of financial health in which expenditures exceed revenue. Revenue Expenditure is the part of government spending that does not result in the production of assets. Example: Salaries, wages, pensions, subsidies, and interest payments etc. The government incurs revenue expenses to meet its operating demands. Reducing the revenue expenditure will certainly help in bridging the gap.

Statement 3 is correct: Subsidies are a part of expenditure and rationalizing them would reduce the deficit.

Statements 2 and 4 are incorrect: Introducing new welfare schemes will most likely result in increasing expenditure and so will expanding industries which would require capital infusion.

 

Question 16

Convertibility of rupee implies:

(a) being able to convert rupee notes into gold.

(b) allowing the value of rupee to be fixed by market forces.

(c) freely permitting the conversion of rupee to other currencies and vice versa.

(d) developing an international market for currencies in India.

Ans: c         

Sub-Theme: Currency Convertibility

Currency convertibility is the ease with which the currency of a country can be freely converted into any other foreign currency or gold at market determined exchange rate. It refers to the ability of rupees to be converted into any foreign currency backed with the exchange rate that prevails at the time of conversion. It allows for easy financial transactions for the export and import of goods and services.

 

Question 17

The problem of international liquidity is related to the nonavailability of:

(a) Goods and services

(b) Gold and silver

(c) Dollars and other hard currencies

(d) Exportable surplus

Ans: c          

Sub-Theme: Basic Economic Concepts

International payments resulting from trade in goods and services are linked to the idea of international liquidity. International liquidity consists of all the resources that are available to the monetary authorities of countries for the purpose of meeting balance of payments deficits. Such liquidity ranges from assets readily available to resources that become available only after extensive negotiation. Gold and those foreign currencies that are accepted everywhere for the settlement of international transactions are the main sources of international liquidity. This problem of international liquidity exists essentially for developing countries.

 

Question 18

In the ‘Index of Eight Core Industries’, which one of the following is given the highest weight?

(a) Coal production

(b) Electricity generation

(c) Fertiliser production

(d) Steel production

Ans: b

Sub-Theme: Index of Industrial Production

  • Index of Industrial Production (IIP):
    • It details the growth of various sectors in an economy such as mining, electricity and manufacturing.
    • The all India IIP is a composite indicator that measures the short-term changes in the volume of production of a basket of industrial products during a given period with respect to that in a chosen base period.
    • It is compiled and published monthly by the Central Statistical Organisation (CSO) six weeks after the reference month ends.
  • Eight Core Sectors:
    • Comprising 40.27% of the weight of items included in the Index of Industrial Production (IIP).
    • The eight core sector industries in decreasing order of their weightage:

Refinery Products > Electricity > Steel

> Coal > Crude Oil > Natural Gas > Cement > Fertilisers.

 

2014

 

Question 1

The main objective of the 12th Five-Year Plan is:

(a) Inclusive growth and poverty growth.

(b) Inclusive and sustainable growth.

(c) Sustainable and inclusive growth to reduce unemployment.

(d) Faster, sustainable and more inclusive growth.

Ans: d

Sub-Theme: Economic Planning

Twelfth FYP (2012-2017):

The Twelfth Five Year Plan (2012-2017) was launched with the objective of faster, sustainable, and more inclusive growth. It was the last five-year plan. Its growth rate target was 8%. Later, the government dissolved the Planning Commission with the NITI Aayog. Twelfth Five-Year Plan objectives:

  • To remove gender and social gaps in school
  • To enhance access to higher
  • To reduce malnutrition among children aged 0–3 years.
  • To provide electricity to all
  • Create 50 million new job opportunities in non-agricultural sectors.

 

Question 2

What does venture capital mean ?

(a) A short-term capital provided to industries

(b) A long-term start-up capital provided to new entrepreneurs

(c) Funds provided to industries at times of incurring losses.

(d) Funds provided for replacement and renovation of industries

Ans: b

Sub-Theme: Business Funding

Option (b) is correct: Venture capital (VC) is a form of private equity and a type of financing that investors provide to startup companies and small businesses that are believed to have long-term growth potential.

Venture Capital:

  • Venture capital (VC) is a form of private equity and a type of financing that investors provide to startup companies and small businesses that are believed to have long- term growth potential.
  • The majority of venture capital is often provided by wealthy individuals, investment banks, and other financial organisations. However, it is not always in the form of money; it can also come in the form of managerial or technological know-how.
  • Venture capital is often given to startups with outstanding growth potential or to businesses that have had rapid growth and seem well-positioned to keep growing.
  • The possibility of above-average profits is an alluring payout, even though it might be risky for investors who put up money.
  • Venture capital is gradually becoming a popular—even necessary—source for obtaining funds for new businesses or projects with a brief working history (under two years), especially if they do not have access to capital markets, bank loans, or other debt instruments.
  • The main downside is that the investors usually get equity in the company, and, thus, a say in company decisions.

 

Question 3

The terms ‘Marginal Standing Facility Rate’ and ‘Net Demand and Time Liabilities’, sometimes appearing in news, are used in relation to:

(a) Banking operations

(b) Communications networking

(c) Military strategies

(d) Supply and demand of agricultural products

Ans: a

Sub-Theme: Banking functions

Marginal standing facility (MSF) is a window for banks to borrow from the Reserve Bank of India in an emergency when interbank liquidity dries up completely.

The difference between a bank’s total demand and time liabilities (deposits) of a bank and its deposits in the form of assets held by another bank is represented by the term “Net Demand and Time Liabilities”.

 

Question 4

What is/are the facility/facilities the beneficiaries can get from the services of Business Correspondent (Bank Saathi) in branchless areas?

  1. It enables the beneficiaries to draw their subsidies and social security benefits in their villages.

  2. It enables the beneficiaries in the rural areas to make deposits and withdrawals.

Select the correct answer using the code given below.

(a) 1 only

(b) 2 only

(c) Both 1 and 2 only

(d) Neither 1 nor 4

Ans: c

Sub-Theme: Banking Functions

Statement 1 is correct: The Reserve Bank of India (RBI) launched the Business Correspondent (BC) Model in 2006 to improve financial inclusion in India. It enables government subsidies and social security benefits to be directly credited to the accounts of the beneficiaries, enabling them to draw the money from the bank saathi or business correspondents in their village itself.

Statement 2 is correct: Banks facilities are essentially brought to the communities via the BC model in their village. They can deposit and withdraw money from their account.

 

Question 5

In the context of the Indian economy, which of the following is/are the purpose/purposes of ‘Statutory Reserve Requirements’?

1. To enable the Central Bank to control the amount of advances the banks can create.

2. To make the people’s deposits with banks safe and liquid.

3. To prevent the commercial banks from making excessive profits.

4. To force the banks to have sufficient vault cash to meet their day-to-day requirements.

Select the correct answer using the code given below.

(a) 1 only

(b) 1 and 2 only

(c) 2 and 3 only

(d) 1, 2, 3 and 4

Ans: a

Sub-Theme: Banking Mechanism/Functions

Statement 1 is correct: When the central bank wants to increase the money supply in the economy, it lowers the reserve ratio. Hence it enables the Central Bank to control the amount of advances the banks can create.

Statement 2 is incorrect: RBI does not requires commercial banks to keep reserves in order to ensure that banks have sufficient assets to draw on when account holders want to be paid. It aims to make banks invest in G-Secs.

Statement 3 is incorrect: Reserve requirements are designed as “precautionary measures” to control the economy and not to stop banks from making “excessive” profit.

Statement 4 is incorrect: Statutory Reserve Requirements are not to force the banks to have sufficient cash to meet their day-to-day requirements.

 

Question 6

With reference to the Union Budget, which of the following is/are covered under Non-Plan Expenditure?

  1. Defence expenditure

  2. Interest payments

  3. Salaries and pensions

  4. Subsidies

Select the correct answer using the code given below.

(a) 1 only

(b) 2 and 3 only

(c) 1, 2, 3 and 4

(d) None

Ans: c         

Sub-Theme: Public Finance

Non-plan Expenditure is largely the revenue expenditure of the government, although it also includes capital expenditure. It covers all expenditure not included in the Plan Expenditure. The majority of non- plan expenses include interest payments, pensions, and statutory transfers to state and union territory governments, interests and debt servicing costs, defence costs, and subsidies. Non-Plan Expenditure constitutes the biggest proportion of the government’s total expenditure. Both revenue and capital expenses are incurred for defence services.

 

Question 7

With reference to Balance of Payments, which of the following constitutes/constitute the Current Account?

  1. Balance of trade

  2. Foreign assets

  3. Balance of invisibles

  4. Special Drawing Rights

Select the correct answer using the code given below.

(a) 1 only

(b) 2 and 3

(c) 1 and 3

(d) 1, 2 and 4

Ans: c         

Sub-Theme: BoP

The balance of payment (BoP) is a record of all monetary transactions made between the residents of one country and the rest of the world. A balance of payments deficit means the nation imports more than it exports. The current account and capital account are the two components that constitute the balance of payments. The current account comprises balance of trade (exports-imports), net remittances, transfers of money as well as transactions in commodities and services (Invisibles). The capital account includes Foreign Direct Investment, Foreign Portfolio Investment, External Commercial Borrowings, SDR. The capital account records all international purchases and sales of assets such as money, stocks, bonds, etc.

 

Question 8

Which of the following organisations brings out the publication known as ‘World Economic Outlook’?

(a) The International Monetary Fund

(b) The United Nations Development Programme

(c) The World Economic Forum

(d) The World Bank

Ans: a

Sub-Theme: IMF Reports

  • The World Economic Outlook (WEO) is a report by the International Monetary Fund (IMF) that analyses key parts of the IMF’s surveillance of economic developments and policies in its member countries. The World Economic Outlook (WEO) presents the IMF’s analysis and projections of global economic developments and classifies their analysis by region and stage of economic development. The World Economic Outlook database is created during the bi-annual WEO exercise, which begins in January and June of each year and results in the April and September/October WEO publication. It is usually prepared twice a year and is used in meetings of the International Monetary and Financial Committee.
  • Other Report published by IMF:

Global Financial Stability Report

 

Question 9

If the interest rate is decreased in an economy, it will:

(a) Decrease the consumption expenditure in the economy.

(b) Increase the tax collection of the government.

(c) Increase the investment expenditure in the  economy.

(d) Increase the total savings in the economy.

Ans: c

Sub-Theme: Money and Banking

Relationship between  investment  and interest rates: The investment curve of the economy serves as another example of expenditure. A decrease in interest rates results in an increase in investment spending. Hence, if the interest rate is decreased in an economy, it will increase the investment expenditure in the economy. Decreased interest rates would ensure the availability of capital for investment expenditure.

 

2013

 

Question 1

Consider the following statements:

  1. Inflation benefits the debtors.

  2. Inflation benefits the bond- holders.

Which of the statements given above is/are correct?

(a) 1 only

(b) 2 only

(c) Both 1 and 2

(d) Neither 1 nor 2

Ans: a

Sub-Theme: Inflation

Statement 1 is correct: Inflation is the rise in prices of goods and services within a particular economy wherein, the purchasing power of consumers decreases, and the value of the cash holdings erode. Inflation redistributes wealth from creditors to debtors i.e. lenders suffer and borrowers benefit out of inflation. Bondholders have lent money (to debtors) and received a bond in return. So he is a lender, he suffers (Debtor benefits from inflation).

Statement 2 is incorrect: The statement has not used specifically the word “inflation- indexed bonds”, hence we cannot say Inflation benefits the bond-holders.

 

Question 2

Which one of the following is likely to be the most inflationary in its effect?

(a) Repayment of public debt

(b) Borrowing from the public to finance a budget deficit

(c) Borrowings from banks to finance a budget deficit

(d) Creating new money to finance a budget deficit

Ans: d

Sub-Theme: Inflation

Option (d) is correct: Borrowing from banks to finance a budget deficit and Borrowing from the public to finance a budget deficit will lead to a decrease in the money supply in the market. Creating new money to finance a budget deficit will have more inflationary effect than Repayment of public debt, as it will lead to an increase in total money supply in the market.

Budget Deficit:

  • It refers to a situation where total expenditure exceeds the total revenue.
  • A tool for raising money to create a budget deficit is known as deficit financing.
  • There are three ways to finance a budget:
    • Creating new currency
    • Borrowing from internal sources like RBI, issuing bonds, etc.
    • Borrowing from External sources like WB, IMF, etc.
  • Inflation won’t occur if the government borrows money from the public by issuing bonds with lower interest In a similar manner, bank loans won’t cause inflation.

 

Question 3

A rise in the general level of prices may be caused by:

  1. An increase in the money supply

  2. A decrease in the aggregate level of output

  3. An increase in the effective demand

Select the correct answer using the codes given below:

(a) 1 only

(b) 1 and 2 only

(c) 2 and 3 only

(d) 1, 2 and 3

Ans: d

Sub-Theme: Inflation

There are several factors that can contribute to a rise in overall prices, including an influx of money into the economy, a decrease in the total amount of goods and services being produced, an uptick in demand for those goods and services, an increase in people’s earnings, and a rapid expansion of the population.

Statement 1 is correct: An increase in the money supply will cause Inflation.

Statement 2 is correct: A decrease in output will cause Inflation.

Statement 3 is correct: An increase in demand will cause Inflation.

Inflation:

  • It is the rise in prices of goods and services within a particular economy wherein, the purchasing power of consumers decreases, and the value of the cash holdings erode.
  • Inflation measures the average price change in a basket of commodities and services over time.
  • The opposite and rare fall in the price index of this basket of items is called ‘deflation’.
  • In India, the Ministry of Statistics and Programme Implementation (MoSPI) measures inflation.
  • Some causes of inflation: Increase in demand, reduction in supply, demand-supply gap, excess circulation of money, increase in input costs, devaluation of currency, rise in wages, among others.
  • In India, inflation is primarily measured by two main indices: Wholesale Price Index (WPI) and Consumer Price Index (CPI). (Refer PYQ for detailed and comparative explanation on WPI and CPI).

NOTE: This is a direct, application based question. Details on inflation have been covered in earlier PYQs.

 

Question 4

Supply of money remaining the same when there is an increase in demand for money, there will be:

(a) A fall in the level of prices

(b) An increase in the rate of interest

(c) A decrease in the rate of interest

(d) An increase in the level of income and employment

Ans: b

Sub-Theme: Inflation

  • If the supply of money remains the same when there is an increase in demand for money, there will be an increase in the rate of interest.
  • This happens so that the banks can draw customers and encourage deposits from them. It will lead to more savings than

 

Question 5

In the context of Indian economy, ‘Open Market Operations’ refers to:

(a) Borrowing by scheduled banks from the RBI

(b) Lending by commercial banks to industry and trade

(c) Purchase and sale of government securities by the RBI

(d) None of the above

Ans: c

Sub-Theme: Monetary Policy

Open Market Operations: It is the purchase and sale of securities by the RBI.

Monetary Policy RBIs

Action

Effect in the Economy
Expansionary Buys Injects Money In
Contractionary Sells Removes Money Out

 

Question 6

Priority Sector Lending by banks in Indian constitutes the lending to:

(a) Agriculture

(b) Micro and small enterprises

(c) Weaker sections

(d) All of the above

Ans: d

Sub-Theme: Priority Sector

The RBI under the PSL, mandates banks to lend a certain portion of their funds to specified sectors: agriculture, Micro, Small and Medium Enterprises (MSMEs), export credit, education, etc.

 

Question 7

Which of the following grants/grant direct credit assistance to rural households?

  1. Regional Rural Banks

  2. National Bank for Agriculture and Rural Development

  3. Land Development Banks

Select the correct answer using the codes given below.

(a) 1 and 2 only

(b) 2 only

(c) 1 and 3 only

(d) 1, 2 and 3

Ans: c

Sub-Theme: Banking Roles and Functions/Credit Assistance

Statement 1 is correct: RRB’s main objective was to provide credit and banking facilities to small and marginal farmers, artisans, labourers, and small entrepreneurs in order to contribute towards the development of the rural economy.

Statement 2 is incorrect: The main agenda behind NABARD’s establishment is to promote agricultural and rural development and provide refinance facilities to Commercial banks, State Cooperative Banks, Central Cooperative Banks, Regional Rural Banks, and Land Development Banks. NABARD does not grant direct credit assistance to rural households.

Statement 3 is correct: To provide long term loans facility to farmers to borrow equipment such as tractors, pump sets, etc. is the objective behind establishment of Land Development Banks.

 

Question 8

The Reserve Bank of India regulates the commercial banks in matters of:

(a) Liquidity of assets

(b) Branch expansion

(c) Merger of banks

(d) Winding-up of banks

Select the correct answer using the codes given below:

(a) 1 and 4 only

(b) 2, 3 and 4 only

(c) 1, 2 and 3 only

(d) 1, 2, 3 and 4

Ans: d

Sub-Theme: Functions of RBI

Statement 1 is correct: Reserve Bank of India is the regulator of banks in India. By issuing mechanisms like SLR, CRR, RBI keeps a check on liquidity of assets of the banks.

Statement 2 is correct: As per Reserve Bank of India’s (RBI) Branch Authorisation Policy, general permission has been granted to domestic Scheduled Commercial Banks (other than RRBs) to open branches by these banks in Tier-1 and Tier-2 centres (centres with population of 50,000 and above) requires prior approval of RBI except in North Eastern States and Sikkim.

Statement     3     is     correct:     Mergers     and acquisitions (M&As) in the banking space may require clearance from fair market watchdog, Competition Commission of India (CCI), as well as the sector regulator, Reserve Bank of India (RBI). Earlier, the indications were that only involuntary mergers and acquisitions, the ones directed by the RBI, would go to the central bank along with the CCI. However, all mergers and acquisitions may now come under both. While the CCI will look at the competition part of such deals, the RBI sees prudential aspects.

Statement 4 is correct: RBI issued orders to wind up a few banks in the recent past. (Sahara India Financial Corp (SIFCL), the para banking arm of Sahara India Pariwar, Siddharth Cooperative Bank, some Banks from Gujarat). Implied from deposit insurance and credit guarantee corporation (DICGC) scheme.

 

Question 9

Consider the following liquid assets:

  1. Demand deposits with the banks

  2. Time deposits with the banks

  3. Savings deposits with the banks

  4. Currency

The correct sequence of these assets in the decreasing order of liquidity is

(a) 1-4-3-2

(b) 4-3-2-1

(c) 2-3-1-4

(d) 4-1-3-2

Ans: d

Sub-Theme: Money Supply

  • Demand deposits with the banks: A demand deposit is money deposited into a bank account with funds that can be withdrawn on-demand at any time. The depositor will typically use demand deposit funds to pay for everyday For funds in the account, the bank or financial institution may pay either a low or zero interest rate on the deposit.
  • Time deposits with the banks: A time deposit is an interest-bearing bank account that has a date of maturity, such as a certificate of deposit (CD). The money in a time deposit must be held for the fixed term to receive the interest in full. Typically, the longer the term, the higher the interest rate that the depositor receives. Time deposits are the least liquid.
  • Savings deposits with the banks: A savings account is an interest-bearing deposit account held at a bank or other financial institution. Though these accounts typically pay a modest interest rate, their safety and reliability make them a great option for parking cash you want available for short- term needs.
  • Liquidity of currency is Demand liabilities will have less liquidity comparatively, as they have to be cleared when demanded. Next will be saving deposits with banks. Currency is the most liquid.

 

Question 10

Which of the following constitute a Capital Account?

  1. Foreign Loans

  2. Foreign Direct Investment

  3. Private Remittances

  4. Portfolio Investment

Select the correct answer using the codes given below:

(a) 1, 2 and 3

(b) 1, 2 and 4

(c) 2, 3 and 4

(d) 1, 3 and 4

Ans: b

Sub-Theme: BoP

Statement 1, 2 and 4 are correct: The capital account shows the net change in the physical or financial asset ownership for a country. It includes Foreign Direct Investment, Portfolio Investment, foreign loans, changes in the reserve account, etc.

Statement 3 is incorrect: Private remittances come under the Current account and not the Capital account.

Balance of Payment (BoP):

  • A systematic record of all economic transactions between the residents of one country with the residents of the other country in a financial year.
  • It consists of balance of trade, balance of current account and capital account.
  • Positive Balance/Trade Surplus: When a country exports more than its imports.
  • Negative Balance/Trade Deficit: When imports are greater than its export.
  • Balance of payments divides transactions in two accounts:
    1. Current account
    2. Capital account
Current Account Capital Account
 

 

Meaning

•   Records imports and exports of visible and invisibles

•   Short term implication transactions

•   Covers only earnings and spending.

•   Excludes any borrowings and lending.

•   Shows capital expenditure and income for country

•   Long term implication transactions

•   Only includes borrowings and lending by a country

 

 

 

Components

•   Visible trade (Export and Import of goods – Merchandise transactions)

•   Invisible trade (Export and Import of services)

•   Unilateral transactions

•   Direct Investment (FDI)

•   Portfolio Investment (FPI)

•   Loans / External commercial borrowing (ECB)

•   Non-resident investment in Bank, Insurance, Pension schemes.

•   RBI’s foreign exchange reserve

 

 

Deficit

•   If the value of imported goods and services is higher than the value of exported ones.

•   Current Account deficit = Trade gap (export – import) + Net current transfers (foreign aid) + Net factor income (Interest, Dividend)

•   When more money is flowing out of a country to acquire assets and rights abroad
 

Surplus

•   If the value of exported goods and services is more than the value of imported ones. •   Money is coming into the country, but these inflows highlight the changes in the ownership of national assets by way of sale or borrowing.
 

Convertibility

•   The removal of restrictions on payments pertaining to the international exchange of goods, services, and factor incomes is known as current account convertibility. •   Capital account convertibility means liberalisation of a country’s capital transactions such as loans and investment.
Current status •   Allowed Full convertibility •   Only Partial convertibility

 

Question 11

Which one of the following groups of items is included in India’s foreign-exchange reserves?

(a) Foreign-currency assets, Special Drawing Rights (SDRs) and loans from foreign countries

(b) Foreign-currency assets, gold holdings of the RBI and SDRs

(c) Foreign-currency assets, loans from the World Bank and SDRs

(d) Foreign-currency assets, gold holdings of the RBI, and loans from the World Bank

Ans: b

Sub-Theme: Forex Reserve

Reserve Bank of India Act and the Foreign Exchange Management Act, 1999 set the legal provisions for governing the foreign exchange reserves. RBI accumulates foreign currency reserves by purchasing from authorised dealers in open market operations. Foreign exchange reserves of India act as a cushion against rupee volatility. India’s foreign exchange reserves comprise foreign currency assets (FCA), gold, special drawing rights (SDRs) and reserve tranche position (RTP) in the International Monetary Fund (IMF).

 

Question 12

To obtain full benefits of the demographic dividend, what should India do?

(a) Promoting skill development

(b) Introducing more social security schemes

(c) Reducing infant mortality rate

(d) Privatisation of higher education

Ans: a

Sub-Theme: Demographic Dividend

Option (a) is correct: Skill development is needed to increase employability of the young population. Government has established ‘Skill India’ as a mission to skill India’s youth and the National Skill Development Corporation (NSDC) with the overall target of skilling/up skilling 500 million people in India by 2022. Option (b) is incorrect: Social security schemes can be used for providing security to the nonworking population like old-age pensions etc. It will not provide any benefit of demographic dividend.

Option (c) is incorrect: Reducing Infant Mortality Rate is one of the Millennium Development Goals, but it does not provide any benefit of demographic dividend.

Option (d) is incorrect: Privatisation of Higher Education will have a negative effect as it will make education costlier thus making it out of reach of a large section of the eligible population.

 

Question 13

Disguised unemployment generally means

(a) Large number of people remain unemployed

(b) Alternative employment is not available

(c) Marginal productivity of labour is zero

(d) Productivity of workers is low

Ans: c

Sub-Theme: Unemployment

Unemployment is a phenomenon that occurs when a person who is capable of working and is actively searching for work is unable to find work. The unemployment rate, which is calculated by dividing the number of jobless persons by the total labour force, is the most widely used indicator of unemployment. Disguised unemployment means that more people are engaged in a job which can be done with less people, normally seen in agriculture where the whole family is It means extra people can be removed without affecting productivity, i.e., their marginal productivity is zero.

 

Question 14

Economic growth in country X will necessarily have to occur if:

(a) There is technical progress in the world economy

(b) There is population growth in X

(c) There is capital formation in X

(d) The volume of trade grows in the world economy

Ans: c

Sub-Theme: Economic Growth and Development

  • Capital Formation is defined as that part of a country’s current output and imports which is not consumed or exported during the accounting period but is set aside as an addition to its stock of capital goods. Capital formation in any country boosts its development as investment activities increase resulting in high production, increase in   per-capita   income, employment generation, the standard of living, poverty reduction.
  • The increase in population has both pros and cons for the economy. If the population is skilled then it will generate more output for the economy. But in case if the economy itself is not in a position to stand still then in such case it will prove to be a burden to the country.
  • The development of new and better technologies for the manufacture of goods and services is referred to as technical advancement. Despite having their advantages, when these technologies enter the market, it requires a heavy investment in training workers and sometimes these technologies replace human beings resulting in unemployment.

The volume of trade may be favourable or non- favourable depending upon the overall trade surplus or deficit.

 

Question 15

The national income of a country for a given period is equal to the:

(a) Total value of goods and services produced by the nationals

(b) Sum of total consumption and investment expenditure

(c) Sum of personal income of all individuals

(d) Money value of final goods and service produced

Ans: d        

Sub-Theme: National Income

Option (d) is correct: The entire monetary/ money value of the goods and services produced on a nation’s territory over the course of a certain amount of time is its national income. Thus, it is the result of every economic activity that takes place throughout the course of a year in every nation. The total amount of income that a nation accrues from all of its economic operations over the course of a year is, in a nutshell, its national income. It is also helpful in determining the progress of the country. It includes wages, interest, rent, profit, received by factors of production like labour, capital, land and entrepreneurship of a nation.

 

Question 16

The balance of payments of a country is a systematic record of:

(a) All import and export transactions of a country during a given period of time, normally a year

(b) Goods exported from a country during a year

(c) Economic transaction between the government of one country to another

(d) Capital movements from one country to another

Ans: a          

Sub-Theme: BoP

Option (a) is correct: The balance of payments (BoP) records the transactions in goods, services and assets between residents of a country with the rest of the world for a specified time period typically a year. The balance of payments (BOP), also known as balance of international payments, summarises all transactions that a country’s individuals, companies, and government bodies complete with individuals, companies, and government bodies outside the country. These transactions consist of imports and exports of goods, services, and capital, as well as transfer payments, such as foreign aid and remittances.

 

Question 17

An increase in the Bank Rate generally indicates that the:

(a) Market rate of interest is likely to fall

(b) Central Bank is no longer making loans to commercial banks

(c) Central Bank is following an easy money policy

(d) Central Bank is following a tight money policy

Ans: d          

Sub-Theme: Monetary Policy/Money and Banking

Option (d) is correct: Bank rate refers to the rate at which the Reserve Bank of India (RBI)  gives loans to banks. An increase in this rate means that RBI is following a tight monetary policy as an increase in rates will lead to decrease in money circulation leading to a

decrease in inflation.

 

Question 18

In India, deficit financing is used for raising resources for:

(a) Economic development

(b) Redemption of public debt

(c) Adjusting the balance of payments

(d) Reducing the foreign debt

Ans: (a)          

Sub-Theme: Budget

Option (a) is correct: Deficit financing is the process of raising money to cover the deficit that is brought on by an excess of spending in comparison to income. By selling bonds or printing new money, the government is borrowing money from the people to close the shortfall. For developing countries like India, higher economic growth is a priority. A higher economic growth requires finances. With the private sector being shy of making huge expenditures, the responsibility of drawing financial resources rests on the government. Many times, neither tax nor non-tax revenues are able to raise enough money. Hence, the deficit is often funded through borrowings or printing new currency notes.

 

2012

 

Question 1

Which of the following can be said to be essentially the parts of ‘Inclusive Governance’?

1. Permitting the Non-Banking Financial Companies to do banking.

2. Establishing effective District Planning Committees in all the districts.

3. Increasing government spending on public health.

4. Strengthening the Mid-day Meal Scheme.

Select the correct answer using the codes given below:

(a) 1 and 2 only

(b) 3 and 4 only

(c) 2, 3 and 4 only

(d) 1, 2, 3 and 4

Ans: c

Sub-Theme: Inclusive growth

Statement 1 is incorrect: Permitting the NBFCs to do banking cannot be a part of Inclusive Governance. The prime motive of Banks is to earn profits; for an inclusive governance concept to be successful, the government has to focus more on support being provided to all citizens irrespective of their economic and social status, rather than allowing entities to churn out profits out of them.

Statement 2 is correct: According to Article 243ZD of the Indian Constitution, the District Planning Committee (DPC) must have been established at the district level to oversee planning at the district and below. The committee in charge of each district shall compile the district’s Panchayat and Municipality development plans and create a draft development plan for the district.

Statement 3 is correct: The Government aims to increase public health expenditure to 2.5% of GDP by 2025. The 15th Finance Commission, the Union Health Minister Dr. Harsh Vardhan, highlighted that the government aims at gradually increasing the public health expenditure to 2.5% of the nation’s GDP by the year 2025.

Statement 4 is correct: The Mid-Day Meal Program or MDMP is a program of, “Nutrition Support to Primary Education” and is considered as a means of mitigating the classroom hunger to all children below the age of 14 years who are enrolled in schools. It was aimed at improving enrolment, attendance, and retention, while simultaneously improving the nutritional status of students in primary classes.

 

Question 2

What is/are the recent policy initiative(s) of Government of India to promote the growth of the manufacturing sector?

  1. Setting up of National Investment and Manufacturing Zones.

  2. Providing the benefit of ‘single window clearance’.

  3. Establishing the Technology Acquisition and Development Fund.

Select the correct answer using codes given below:

(a) 1 only

(b) 2 and 3 only

(c) 1 and 3 only

(d) 1, 2 and 3

Ans: d

Statement 1 is correct: The National manufacturing Policy provides for the promotion of clusters and aggregation, especially through the creation of National Investment and Manufacturing Zones (NIMZs).

Statement 2 is correct: Initiating the process of simplification and rationalization of state-level business regulations and developing single-window clearance.

Statement 3 is correct: Advisory to States on simplification & rationalization of business regulations and skill Development. Constitution of the approval/monitoring mechanism under the policy. Scheme prepared under NMP on Technology Acquisition and Development Fund (TADF).

 

Question 3

In India, in the overall index of Industrial Production, the Indices of Eight Core Industries have a combined weight of 90%. Which of the following are among those Eight Core industries?

  1. Cement

  2. Fertilizer

  3. Natural Gas

  4. Refinery products

  5. Textiles

Select the correct answer using the codes given below:

(a) 1 and 5 only

(b) 2, 3 and 4 only

(c) 1, 2, 3 and 4 only

(d) 1, 2, 3, 4 and 5

Ans: c

Sub-Theme: Core Industry

Eight Core Industries:

  • Eight Core Industries measures combined and individual performance of production in selected eight core industries viz. Coal, Crude Oil, Natural Gas, Refinery Products, Fertilizers, Steel, Cement and Electricity. The Eight Core Industries comprise 27% (2021) of the weight of items included in the Index of Industrial Production (IIP).
  • These industries have a major impact on general economic activities and also industrial activities.
  • They significantly impact most other industries as well. The capital basis of the economy is represented by the core
  • The IIP provides the economic growth rates for various industrial categories over a certain time period.

 

Question 4

Despite having large reserves of coal, why does India import millions of tons of coal?

1. It is the policy of India to save its own coal reserves for the future, and import it from other countries for the present use.

2. Most of the power plants in India are coal-based and they are not able to get sufficient supplies of coal from within the country.

3. Steel companies need a large quantity of coking coal which has to be imported.

Which of the statements given above is/are correct?

(a) 1 only

(b) 2 and 3 only

(c) 1 and 3 only

(d) 1, 2 and 3

Ans: b

Sub-Theme: Coal Sector

Statement 1 is incorrect: There is no such policy of India to save its own coal reserves for the future and import it from other countries for the present use.

Statement 2 is correct: Coal based power plants, cement plants, captive power plants, sponge iron plants, industrial consumers and coal traders are importing non-coking coal.

Statement 3 is correct: The production of steel requires 0.8 tonnes of coking coal to produce one tonne of Steel. And about 85% of the coking coal requirement of the domestic steel industry is presently being met through imports.

Additional Information:

Import of Coal:

  • According to the current import policy, consumers themselves may freely import coal (under Open General Licence) based on their own requirements and business
  • Steel Authority of India Limited (SAIL) and other steel-making facilities import coking coal primarily to fill the gap between demand and supply domestically and to raise the level of production quality. About 85% of the coking coal requirement of the domestic steel industry is presently being met through imports.
  • Coal based power plants, cement plants, captive power plants, sponge iron plants, industrial consumers and coal traders are importing non-coking coal.
  • Coke is imported mainly by pig-Iron manufacturers and Iron & Steel sector consumers using mini-blast furnace.

 

Question 5

Which of the following measures would result in an increase in the money supply in the economy?

  1. Purchase of government securities from the public by the Central Bank.

  2. Deposit of currency in commercial banks by the public.

  3. Borrowing by the government from the Central bank.

  4. Sale of government securities to the public by the Central Bank.

Select the correct answer using the codes given below:

(a) 1 only

(b) 2 and 4 only

(c) 1 and 3

(d) 2, 3 and 4

Ans: c

Sub-Theme: Money Supply

Statement 1 is correct: The Central Bank pays investors holding bonds. If the Central  Bank  buys  Government  securities (or corporate bonds) people who are holding the bonds have more money to spend. Banks see illiquid assets become liquid. Therefore, in certain circumstances, this can lead to an increase in the money supply.

Statement 2 is incorrect: Deposit of currency in commercial banks by the public does not increase, instead decreases the supply of money in the market.

Statement 3 is correct: Borrowing by the government from the Central Bank leads to the release of money to the Central Bank which eventually releases money in the market via loans etc.

Statement 4 is incorrect: Sale of the government securities to the public by the Central Bank leads to more flow of money in the market.

 

Question 6

The Reserve Bank of India (RBI) acts as a bankers’ bank. This would imply which of the following?

  1. Other banks retain their deposits with the RBI

  2. The RBI lends funds to the commercial banks in times of need.

  3. The RBI advises the commercial banks on monetary matters.

Select the correct answer using the codes given below:

(a) 2 and 3 only

(b) 1 and 2 only

(c) 1 and 3 only

(d) 1, 2 and 3

Ans: d

Sub-Theme: Role and Functions of RBI

Statement 1 is correct: The Reserve Bank acts as the Banker, Agent, and Adviser to the Government of India and states. It performs the same functions for the other commercial banks as the other banks ordinarily perform for their customers.

Statement 2 is correct: RBI lends money to all the commercial banks of the country.

Statement 3 is correct: The RBI is also providing advisory functions to banks on financial matters and also on general economic problems.

 

Question 7

The basic aid of Lead Bank Scheme is that:

(a) Big banks should try to open offices in each district.

(b) There should be stiff competition among the various nationalized banks.

(c) Individual banks should adopt a particular district for intensive development.

(d) All the banks should make intensive efforts to mobilise deposits.

Ans: c

Sub-Theme: Lead Bank Scheme

The basic aid of the Lead Bank Scheme is that the individual banks should adopt a particular district for intensive development.

Lead Bank Scheme:

  • RBI introduce on the recommendation of the Gadgil Study Group and Banker’s
  • Introduced towards the end of 1969, envisages the assignment of lead roles to individual banks (both in the public sector and private sector) for the districts allotted to them.
  • Objective: To help in removing regional imbalances through appropriate credit deployment and to extend banking facilities to unbanked areas.

 

Question 8

Under which of the following circumstances may ‘capital gains’ arise?

  1. When there is an increase in sales of product.

  2. When there is a natural increase in the value of the property owned.

  3. When you purchase a painting, there is a growth in its value due to an increase in its popularity.

Select the correct answer using the codes given below:

(a) 1 only

(b) 2 and 3 only

(c) 2 only

(d) 1, 2 and 3

Ans: b

Sub-Theme: Taxation

Option (b) is correct: ‘Capital gains’ occur when the value of an asset, such as a property or a painting, increases over time, resulting in a profit when it is sold. This increase in value can happen naturally or due to a rise in demand or popularity, as in the case of a painting.

Statement 1 is incorrect: Increase in sales of products not counted under capital gains. Hence, statement 1 is wrong.

Statement 2 is correct: A capital gain is the increase in a capital asset’s value and is realised when the asset is sold.

Statement 3 is correct: Capital gains apply to any type of asset, including investments and those purchased for personal use.

Capital Gain:

  • The increase in value realised upon selling a capital asset is referred to as a capital gain. All assets, including investments and those bought for personal use, are subject to capital gains.
  • The gain may be short-term (one year or less) or long-term (more than one year) and must be claimed on income taxes. Any profit or gain that arises from the sale of a ‘capital asset’ is a capital gain.
  • Gains from the sale of capital assets are subject to the capital gains tax. This gain or profit is charged to tax in the year in which the transfer of the capital asset takes place.
  • ‘Capital Gains’ arise when,
    • Whenever the property’s value naturally increases.
    • A product’s value increases when you buy it because of an uptick in popularity.

The following are not included under capital assets:

  • Any stock, consumables or raw materials that are held for the purpose of business or
  • Goods such as clothes or furniture or paintings which are held for personal use.
  • Land or there is natural increase in
  • Special bearer bonds were issued in
  • Gold bonuses issued by the Central
  • Gold deposit

 

Question 9

Which of the following would include Foreign Direct Investment in India?

  1. Subsidiaries of foreign companies in India.

  2. Majority foreign equity holding in Indian companies.

  3. Companies exclusively financed by foreign companies.

  4. Portfolio investment.

Select the correct answer using the codes given below:

(a) 1, 2, 3 and 4

(b) 2 and 4 only

(c) 1 and 3 only

(d) 1, 2 and 3 only

Ans: d          

Sub-Theme: Foreign Investment

Statements 1, 2 and 3 are correct: Foreign Direct Investment (FDI): is the investment through capital instruments by a person resident outside India (a) in an unlisted Indian company; or (b) in 10 percent or more of the post issue paid-up equity capital on a fully diluted basis of a listed Indian company. FDIs include Subsidiaries of foreign companies in India, Majority foreign equity holding in Indian companies, Companies exclusively financed by foreign companies, Non-debt Financial resources, Day to day operations of the company. Etc.

Statement 4 is incorrect: Foreign Portfolio Investment (FPI): is any investment made by a person resident outside India in capital instruments where such investment is (a) less than 10 percent of the post issue paid-up equity capital on a fully diluted basis of a listed Indian company or (b) less than 10 percent of the paid up value of each series of capital instruments of a listed Indian company. Portfolio investment comes under FPI.

 

Question 10

Consider the following statements:

The price of any currency in international market is decided by the

  1. World Bank

  2. Demand for goods/services provided by the country concerned

  3. Stability of the government of the concerned country

  4. Economic potential of the country in question

Which of the statements given above are correct?

(a) 1, 2, 3, and 4

(b) 2 and 3 only

(c) 3 and 4 only

(d) 1 and 4 only

Ans: b          

Sub-Theme: Currency Value/IMF

Statement 1 is incorrect: The World Bank is important to the source of financial and technical assistance to developing countries around the world. It has no relation to the price of the currency in the International Market.

Statement 2 is correct: Demand and supply factors have a role in setting the price of every currency. A currency’s demand is influenced by two things: its exports to other nations and the investments people desire to make in that currency.

Statement 3 is correct: The stability of the government is a very important factor as an unstable government may not be able to take effective economic decisions which will, in turn, affect export and import.

Statement 4 is incorrect: The value of any currency on the international market has no bearing on the economic capacity of a nation.

 

Question 11

How does the National Rural Livelihood Mission seek to improve livelihood options of rural poor?

  1. By setting up a large number of new manufacturing industries and agribusiness centres in rural areas.

2. By strengthening ‘self-help groups’ and providing skill development.

3. By supplying seeds, fertilisers, diesel pump-sets and micro-irrigation equipment free of cost to farmers.

Select the correct answer using the codes given below:

(a) 1 and 2 only

(b) 2 only

(c) 1 and 3 only

(d) 1, 2 and 3

Ans: b

Sub-Theme: Government Schemes and Policies Statement 2 is correct and Statement 1 and 3 are incorrect:

  • Deen Dayal Antyodaya Yojana – National
  • Livelihoods Mission (NRLM):
    • Launched: by the Ministry of Rural Development (MoRD), Government of India in June 2011 as a restructured version of Swarna Jayanti Gram Swarozgar Yojna (SGSY).
    • Aims: To build strong institutional frameworks for the rural poor that will enable them to improve their access to financial services and raise household incomes through improvements to their sustainable means of subsistence.
  • Objective: To Cover 7 crore rural poor families throughout 600 districts, 6000 blocks, 2.5 lakh Gram Panchayats, and 6 lakh villages through self-managed Self-Help Groups (SHGs) and federal institutions, and assist them for livelihood collectives during an 8–10 year timeframe.

 

Question 12

With reference to the National Rural Health Mission, which of the following are the jobs of ‘ASHA’, a trained community health worker?

  1. Accompanying women to the health facility for antenatal care check-up.

  2. Using pregnancy test kits for early detection of pregnancy.

  3. Providing information on nutrition and immunisation.

  4. Conducting the delivery of a baby.

Select the correct answer using the codes given below:

(a) 1, 2 and 3 only

(b) 2 and 4 only

(c) 1 and 3 only

(d) 1, 2, 3 and 4

Ans: a

Sub-Theme: Government Schemes and Policies

Statements 1 and 3 are correct: ASHA will mobilise the community and facilitate them in accessing health and health related services such as providing immunization, Antenatal Check-up (ANC), Post Natal Check-up supplementary nutrition, etc.

Statement 2 is correct: Empowered with knowledge and a drug-kit to deliver first- contact healthcare along with identifying and registering new pregnancies, births, and deaths.

Statement 4 is incorrect: Delivery of a baby is being conducted by a professional medical practitioner not by ASHA worker.

NOTE: By general understanding, we know that ASHAs are not paramedics and not trained in providing “delivery of a baby” as it is conducted by a professional medical practitioner. However, supporting health service delivery through home visits, first- aid and immunisation sessions are done by ASHA workers. With this information, we can eliminate option 4.

 

Question 13

The endeavour of ‘Janani Suraksha Yojana’ Programme is

  1. To promote institutional

  2. To provide monetary assistance to the mother to meet the cost of delivery.

  3. To provide for wage loss due to pregnancy and

Which of the statements given above is/are correct?

(a) 1 and 2 only

(b) 2 only

(c) 3 only

(d) 1, 2 and 3

Ans: a          

Sub-Theme: Government Schemes and Policies

Statement 1 is correct: Janani Suraksha Yojana (JSY) is a safe motherhood intervention under the National Rural Health Mission (NRHM). The objective is to reduce maternal and neonatal mortality by promoting institutional delivery among the poor pregnant women.

Statement 2 is correct: JSY is a 100 % centrally sponsored scheme and it integrates cash assistance with delivery and post- delivery care.

Statement 3 is incorrect: To provide compensation for the wage loss in terms of cash incentives so that the woman can take adequate rest before and after delivery of the first living child comes under Pradhan Mantri Matru Vandana Yojana.

 

Question 14

Consider the following:

  1. Hotels and restaurants

  2. Motor transport undertakings

  3. Newspaper establishments

  4. Private medical institutions

The employees of which of the above can have the ‘Social Security’ coverage under Employees’ State Insurance Scheme?

(a) 1, 2 and 3 only

(b) 4 only

(c) 1, 3 and 4 only

(d) 1, 2, 3 and 4

Ans: d          

Sub-Theme: Government Schemes and Policies

Statements 1, 2, 3 and 4 are correct: Employees State Insurance Scheme •    The Employees’ State Insurance Act of 1948 is a piece of social security legislation that covers medical care and financial benefits in the events of sickness, maternity, disability, or worker death as a result of work-related injuries.

  • The Act applies, in the first instance, to non- seasonal factories employing 10 or more persons.
  • The provisions of the Act are being extended area wise by stages.
  • An enabling clause in the Act gives the “appropriate government” the authority to apply the Act’s provisions to various types of institutions, including industrial, commercial, agricultural, and other types.
  • In accordance with these provisions, the majority of State governments have expanded the application of the Act’s provisions to new classes of businesses, including shops, hotels, restaurants, cinemas, including preview theatres, road-motor transportation undertakings, and newspaper businesses with 20 or more employable workers.

 

Question 15

If the National Water Mission is properly and completely implemented, how will it impact the country?

1. Part of the water needs of urban areas will be met through recycling of waste-water.

2. The water requirements of coastal cities with inadequate alternative sources of water will be met by adopting appropriate technologies that allow for the use of ocean water.

3. All the rivers of Himalayan origin will be linked to the rivers of peninsular India.

4. The expenses incurred by farmers for digging bore-wells and for installing motors and pump- sets to draw ground-water will be completely reimbursed by the Government.

Select the correct answer using the codes given below:

(a) 1 only

(b) 1 and 2 only

(c) 3 and 4 only

(d) 1, 2, 3 and 4

Ans: b          

Sub-Theme: Water conservation

Statements 1 and 2 are correct: National Water Mission is one of the eight missions launched under the National Action Plan on Climate Change (NAPCC) for combating the threats of global warming. The main objective of NWM is “conservation of water, minimising wastage and ensuring its more equitable distribution both across and within States through integrated water resources development and management”. It will seek to ensure that a considerable share of the water needs of urban areas are met through recycling of wastewater and ensuring that the water requirements of coastal cities with inadequate alternative sources of water are met through adoption of new and appropriate technologies such as low temperature desalination technologies that allow for the use of ocean water.

Statement 3 is incorrect: All the rivers of Himalayan origin will be linked to the rivers of peninsular India falling under Interlinking of Rivers Programme, where linking different surplus rivers of the country with deficient rivers take place.

Statement 4 is incorrect: Reimbursement of complete expenses on digging bore wells and motors and pump installations sound absurd, as the government would not encourage something which will deplete the groundwater further.

 

Question 16

How do District Rural Development Agencies (DRDAs) help in the reduction of rural poverty in India?

1. DRDAs act as Panchayati Raj Institutions in certain specified backward regions of the

2. DRDAs undertake area-specific scientific study of the causes of poverty and malnutrition and prepare detailed remedial measures.

3. DRDAs secure inter-sectoral and interdepartmental coordination and cooperation for effective implementation of anti-poverty programmes.

4. DRDAs watch over and ensure effective utilisation of the funds intended for anti-poverty

Which of the statements given above is/are correct?

(a) 1, 2 and 3 only

(b) 3 and 4 only

(c) 4 only

(d) 1, 2, 3 and 4

Ans: b          

Sub -Theme: Poverty Alleviation/Rural Development/Inclusive Growth

Statement 1 is incorrect: The DRDAs are expected to coordinate effectively with the Panchayati Raj Institutions (PRIs). Under no circumstances will they perform the functions of PRIs.

Statement 2 is incorrect: No such provisions are there for DRDAs to undertake an area- specific scientific study of the causes of poverty and malnutrition and prepare detailed remedial measures.

Statement 3 is correct: The role of the DRDA is in terms of planning for effective implementation of anti-poverty programmes; coordinating with other agencies- Governmental, non-Governmental, technical and financial for successful programme implementation; etc.

Statement 4 is correct: DRDA will watch over and ensure effective utilisation of the funds intended for anti-poverty programmes, it will need to develop a far greater understanding of the processes necessary for poverty alleviation/ eradication.

 

Question 17

The Multi-dimensional Poverty Index developed by ‘Oxford Poverty and Human Development’ initiative with UNDP support covers which of the following?

1. Deprivation of education, health, assets and services at household level.

  1. Purchasing power parity at national level

  2. Extent of budget deficit and GDP growth rate at national level.

Select the correct answer using the codes given below:

(a) 1 only

(b) 2 and 3 only

(c) 1 and 3 only

(d) 1, 2 and 3

Ans: a

Sub-Theme: Reports and Indices

    • Global Multidimensional Poverty Index is a key international resource that measures acute multidimensional poverty across more than 100 developing countries. It was first launched in 2010 by the Oxford Poverty and Human Development Initiative (OPHI) and the Human Development Report Office of the UNDP.
    • Poverty is often defined by one-dimensional measures usually based on But no single indicator can capture the multiple dimensions of poverty.
    • Multidimensional poverty includes: poor health, lack of education, inadequate living standards, disempowerment, poor quality of work, the threat of violence, and living in areas that are environmentally hazardous, among others.

 

2011

 

Question 1

In the context of Indian economy, consider the following statements:

  1. The growth rate of GDP has steadily increased in the last five years.

  2. The growth rate in per capita income has steadily increased in the last five years.

Which of the statements given above is/are correct?

(a) 1 only

(b) 2 only

(c) Both 1 and 2

(d) Neither 1 nor 2

Ans: c

Sub-Theme: Sustainable Agriculture

Drip Irrigation:

  • It is a form of irrigation where water is applied near the plant root through emitters or drippers, on or below the soil surface, at a low rate varying from 2-20 litres per Frequent watering helps to maintain the optimum level of soil moisture.
  • The most effective irrigation technique is drip irrigation, which may be used for a wide range of crops, including vegetables, orchard crops, flowers, and plantation

Advantages of Drip Irrigation:

  • Maximum use of available
  • No water being available to
  • Maximum crop
  • High efficiency in the use of
  • Weed growth is reduced and restricts the population of potential hosts.
  • Low labour and relatively low operation
  • No soil
  • Improved infiltration in soil of low
  • Ready adjustment to sophisticated automatic control.
  • No runoff of fertilizers into
  • Less water is lost to evaporation than during surface irrigation.
  • Improves seed
  • Decreased tillage

Disadvantages of Drip Irrigation:

  • Sensitivity to clogging
  • Moisture distribution problem
  • Salinity hazards
  • High cost compared to
  • For design, installation, and operation, high competence is needed.

 

Question 2

India has experienced persistent and high food inflation in the recent past. What could be the reasons?

1. Due to a gradual switchover to the cultivation of commercial crops, the area Under cultivation of food grains has steadily decreased in the last five years by about 30%.

2. As a consequence of increasing incomes, the consumption patterns of the people have undergone a significant change.

3. The food supply chain has structural constraints.

Which of the statements given above are correct?

(a) 1 and 2 only

(b) 2 and 3 only

(c) 1 and 3 only

(d) 1, 2 and 3

Ans: b

Sub-Theme: Inflation/Types of Inflation

Inflation:

  • It is the rise in prices of goods and services within a particular economy wherein, the purchasing power of consumers decreases, and the value of the cash holdings erode.
  • Inflation measures the average price change in a basket of commodities and services over time.
  • The opposite and rare fall in the price index of this basket of items is called ‘deflation’.
  • In India, the Ministry of Statistics and Programme Implementation (MoSPI) measures inflation.
  • Some causes of inflation: Increase in demand, reduction in supply, demand-supply gap, excess circulation of money, increase in input costs, devaluation of currency, rise in wages, among others.

COBWEB PHENOMENON

  • Explains large scale fluctuations in the prices of Pulses in the Indian Market.
  • If prices were higher in the previous year, more farmers would sow pulses in the current year leading to its over-production and subsequent decline in the prices.
  • The lower prices in the current year disincentive the farmers from growing crops in the next cropping season, leading to underproduction and subsequent increase in the prices.

NOTE: This particular question came back in 2011, thus all the facts and data w.r.t. the question becomes irrelevant in present times. However, covering the Inflation topic holistically still stands very important both in terms of UPSC Prelims and Mains perspective.

 

Question 3

Why is the Government of India disinvesting its equity in the Central Public Sector Enterprises (CPSEs)?

1. The Government intends to use the revenue earned from the disinvestment mainly to pay back the external debt.

2. The Government no longer intends to retain the management control of the CPSEs.

Which of the statements given above is/are correct?

(a) 1 only

(b) 2 only

(c) Both 1 and 2

(d) Neither 1 nor 2

Ans: d

Sub-Theme: Disinvestment

Statement 1 is incorrect: External debt is the portion of a country’s debt that is borrowed from foreign lenders, including commercial banks, governments, or international financial institutions. The Government does not necessarily intend to use the revenue earned from the disinvestment mainly to pay back the external debt.

Statement 2 is incorrect: From time to time, the Government takes the decision to disinvest the CPSEs but largely it decides to retain the management control on a case to case basis.

Central Public Sector Enterprises: These are the businesses where the Central Government or other CPSEs directly own 51% or more. The Ministry of Heavy Industries and Public Enterprises is in charge of managing these. Additionally, these are divided into strategic and non- strategic CPSE.

 

Question 4

With what purpose is the Government of India promoting the concept of “Mega Food Parks”?

  1. To provide good infrastructure facilities for the food processing industry.

  2. To increase the processing of perishable items and reduce wastage.

3. To provide emerging and eco-friendly food processing technologies to entrepreneurs.

Select the correct answer using the codes given below:

(a) 1 only

(b) 1 and 2 only

(c) 2 and 3 only

(d) 1, 2 and 3

Ans: d

Sub-Theme: Food Processing

Mega Food Park Scheme 2008-09:

  • It was introduced in 2008–09 with the goal of significantly boosting the food processing industry by increasing value and lowering food waste at every point in the supply chain, with a focus on perishables. The Mega Food Park Scheme is being carried out in the nation by the Ministry of Food Processing Industries.
  • Through a cluster-based approach, mega food parks build cutting-edge infrastructure facilities for food processing along the value chain from farm to market with strong forward and backward connections.
    • Aim: To provide a method for connecting agricultural production to the market by bringing together producers, processors, and retailers in order to maximise value addition, reduce wastage, boost farmers’ incomes, and create jobs, particularly in the rural sector.
    • Approach: The “Cluster” approach- based Scheme plans to build state- of-the-art support infrastructure in a clearly defined agri/horticultural zone to facilitate the establishment of contemporary food processing units on the industrial plots offered in the park with a well-established supply chain.
    • Components: A Mega Food Park normally includes cold chain, collection centres, primary processing centres, central processing centres, and about 25 to 30 fully completed plots for entrepreneurs to build food processing units.
    • Financial Assistance: The central government provides financial assistance up to Rs. 50 Crore per Mega Food Park (MFP) project. The MFP project is implemented by a Special Purpose Vehicle (SPV) which is a Body Corporate registered under the Companies Act, 2013.
    • Present Status: Presently, 22 Mega Food Parks are operational. It is in line with the ‘Make in India’ and ‘Atma Nirbhar Bharat’ vision of the Government of India.

 

Question 5

Why is the offering of “teaser loans” by commercial banks a cause of economic concern?

  1. The teaser loans are considered to be an aspect of subprime lending and banks may be exposed to the risk of defaulters in future.

  2. In India, the teaser loans are mostly given to inexperienced entrepreneurs to set up manufacturing or export units.

Which of the statements given above is/are correct?

(a) 1 only

(b) 2 only

(c) Both 1 and 2

(d) Neither 1 nor 2

Ans: a

Sub-Theme: NPA

Statement 1 is correct: Teaser loans are considered an aspect of subprime lending.

Statement 2 is incorrect: Teaser loans are generally offered/provided to entrepreneurs and new homeowners. Experience is not a criterion.

Teaser Loan:

  • A teaser loan is any loan that offers a lower interest rate for a fixed amount of time as a purchase incentive.
  • Teaser loans are a common promotional offering for loan issuers that frequently attract a wide range of borrowers.
  • Low-introductory-rate credit cards and adjustable-rate mortgages are examples of common teaser loans. State Bank of India pioneered the teaser loan concept in home loans in 2009.
  • Teaser loans are considered an aspect of subprime lending.
  • Teaser loans are generally offered/provided to entrepreneurs and new homeowners.
  • Experience is not a
  • Subprime lending is the practice of lending to borrowers with a low credit rating that may be exposed to the risk of default in

 

Question 6

In India, which of the following have the highest share in the disbursement of credit to agriculture and allied activities?

(a) Commercial Banks

(b) Cooperative Banks

(c) Regional Rural Banks

(d) Microfinance Institutions

Ans: a

Sub-Theme: Credit disbursement

  • In India, commercial banks have the highest share in the disbursement of credit to agriculture and allied activities.
  • The commercial banks disburse around 60% credit followed by cooperative banks around 30% and RRB and others.

 

Question 7

Which of the following can aid in furthering the Government’s objective of inclusive growth?

  1. Promoting Self-Help

  2. Promoting Micro, Small and Medium

  3. Implementing the Right to Education

Select the correct answer using the codes given below:

(a) 1 only

(b) 1 and 2 only

(c) 2 and 3 only

(d) 1, 2 and 3

Ans: d

Sub-Theme: Inclusive Growth

Statement 1 is correct: The more the Self- Help Groups, the better the financial inclusion. Statement 2 is correct: MSMEs are an important sector for the Indian economy and have contributed immensely to the country’s socio-economic development. It can help in financial inclusion.

Statement 3 is correct: The RTE Act mandates 25% reservation for disadvantaged sections of the society where disadvantaged groups include: SCs and STs. Education can help uplift the backward and downtrodden society.

Inclusive Growth:

  • Inclusive growth is economic growth that is distributed fairly across society and creates opportunities for all-as defined by OECD (Organisation for Economic Co-operation and Development).
  • It refers to the impoverished having access to essential services in health and education. It entails promoting equality of opportunity and empowering individuals via skill development and education.
  • It also includes a growth process that is environmentally friendly, strives for good governance, and aids in the development of a society that is gender aware.
  • Self Help Groups (SHGs) are working at the grassroots level in India right now, especially in rural India, on the tenets of mutual aid, solidarity, and shared responsibility. The more the Self Help Groups, the better the financial inclusion.
  • The Government of India has introduced MSME or Micro, Small, and Medium Enterprises in agreement with the Micro, Small and Medium Enterprises Development (MSME) Act of 2006.
  • MSMEs are an important sector for the Indian economy and have contributed immensely to the country’s socio-economic development. It not only generates employment opportunities but also works hand-in-hand towards the development of the nation’s backward and rural It can help in financial inclusion.

The RTE Act aims to provide primary education to all children aged 6 to 14 years. The act mandates 25% reservation for disadvantaged sections of the society where disadvantaged groups include: SCs and STs. Education can help uplift the backward and downtrodden society.

 

Question 8

Microfinance is the provision of financial services to people of low-income groups. This includes both the consumers and the self-employed. The service/ services rendered under microfinance is/are:

  1. Credit facilities

  2. Savings facilities

  3. Insurance facilities

  4. Fund Transfer facilities

Select the correct answer using the codes given below the lists:

(a) 1 only

(b) 1 and 4 only

(c) 2 and 3 only

(d) 1, 2, 3 and 4

Ans: d

Sub-Theme: Microfinance/NBFCs

Microfinance:

  • A type of banking service provided to unemployed or low-income individuals or groups to help them to come out of poverty through income generating activities.
  • Besides SCBs, RRBs, small finance banks, Co-operative banks, NBFCs and NBFC-MFIs also provide microfinance services.
  • Microfinancing is offering financial services to low-income populations. These services include micro-loans, micro-savings and
  • Microfinance institutions (MFIs) are financial companies that provide small loans to people who do not have any access to banking facilities. The definition of “small loans” varies between countries. In India, all loans that are below Rs.1 lakh can be considered microloans.
  • In most cases the so-called interest rates are lower than those charged by normal banks, certain rivals of this concept accuse microfinance entities of creating gain by manipulating poor people’s money.

 

Question 9. Which one of the following is not a feature of “Value Added Tax”?

(a) It is a multi-point destination-based system of taxation

(b) It is a tax levied on value addition at each stage of transaction in the production distribution chain

(c) It is a tax on the final consumption of goods or services and must ultimately be borne by the consumer

(d) It is basically subject of the Central Government and the State Governments are only a facilitator for its successful implementation

Ans: d

Sub-Theme: Indirect Taxes

Option (d) is correct: A value-added tax (VAT) is a type of tax on consumption that is levied on a product whenever its value increases during its production and final sale. The VAT paid by the consumer is the cost of the product minus the VAT on the materials already taxed during its production. For instance, a TV manufacturer pays VAT on all the supplies it uses to make the TV, and the buyer pays the VAT on the final sale price of the TV.

  • Value Added Tax is a ‘tax on sale or purchase of goods within a State e., it is a State Subject by virtue of Entry 54 of State List of the Seventh Schedule of the Constitution of India. Since VAT/Sales tax is a “State subject”, the Central Government has been playing the role of a facilitator for successful implementation of VAT.
  • VAT is a kind of tax levied on the sale of goods and services when these commodities are ultimately sold to the VAT is an integral part of the GDP of any country.
  • VAT is a multi-stage tax that is levied at each step of production of goods and services which involves sale/purchase. Any person earning an annual turnover of more than Rs.5 lakh by supplying goods and services is liable to register for VAT payment. Value-added tax or VAT is levied both on local as well as imported goods.

 

Question 10

In terms of economy, the visit by foreign nationals to witness the XIX common Wealth Games in India amounted to:

(a) Export

(b) Import

(c) Production

(d) Consumption

Ans: a          

Sub-Theme: International Trade

Option (a) is correct: Exports are goods and services that are produced in one country used by another. In this case, Foreign nationals visiting India will buy Indian goods and use Indian services and Indian companies will get foreign currency.

  • Imports are goods and services bought in one country produced by another. In this case foreign nationals will import goods from India after watching the XIX Commonwealth Games.
  • Production is the process of manufacturing goods while Consumption is the usage of manufactured goods.

 

Question 11

Consider the following actions which the government can take:

  1. Devaluing the domestic

  2. Reduction in the export

  3. Adopting suitable policies which attract greater FDI and more funds from FIIs.

Which of the above action(s) can help in reducing

the current account deficit?

(a) 1 and 2

(b) 2 and 3

(c) 3 only

(d) 1 and 3

Ans: d         

Sub-Theme: BoP

Statement 1 is correct: The Government can reduce the substantial current account deficit by increasing exports or by decreasing imports which can be through import restrictions, quotas, or duties or by subsidising exports. Manipulating the exchange rate for cheaper exports tends to increase balance of payments through devaluing of domestic currency.

Statement 2 is incorrect: Reduction in the export subsidy might impact negatively.

Statement 3 is correct: Current account deficit can be lowered by promoting an investor friendly environment. In a capital account, capital inflows result from instruments like foreign investment, loans and banking capital, etc.

 

Question 12

Regarding the International Monetary Fund, which one of the following statements is correct?

(a) It can grant loans to any country

(b) It can grant loans to only developed countries

(c) It grants loans to only member countries

(d) It can grant loans to the central bank of a country

Ans: c        

Sub-Theme: IMF

Option (c) is correct: The International Monetary Fund (IMF) is an international organisation that aims to reduce global poverty, encourage international trade, and promote financial stability and economic growth. It was created in 1945 and is based in Washington, DC. There are a total of 190 member countries, each of which is represented on the group’s board. This representation is based on how important its financial position is in the world, so countries with a stronger financial position have a greater say in the organisation than countries with a weaker financial position. Instead of providing money to fund specific projects, the IMF supports the economy of its member nations that are experiencing balance of payments issues.

 

Question 13

Both Foreign Direct Investment (FDI) and Foreign Institutional Investor (FII) are related to investment in a country. Which one of the following statements best represents an important difference between the two?

(a) FII helps bring better management skills and technology, while FDI only brings in capital

(b) FII helps in increasing capital availability in general, while FDI only targets specific sectors

(c) FDI flows only into the secondary market, while FII targets primary market

(d) FII is considered to be more stable than FDI

Ans: b        

Sub-Theme: Foreign Investment

Option (b) is correct: An investment that a parent firm makes in another nation is known as a foreign direct investment (FDI). On the contrary, Foreign Institutional Investor (FII) is an investment made by an investor in the markets of a foreign nation. FDI targets the primary market whereas FII flows only into the secondary market. Foreign Direct Investment targets a specific enterprise and brings better management skills and technology. While the FII increases capital availability in general. Compared to Foreign Institutional Investors, foreign direct investment is regarded as being more stable.

 

Question 14

Economic growth is usually coupled with:

(a) Deflation

(b) Inflation

(c) Stagflation

(d) Hyperinflation

Ans: b          

Sub-Theme: Economic Growth/Inflation

Option (b) is correct: Economic growth: Increased consumer disposable income as a result of economic growth raises both the total demand for goods as well as the quantity that is readily available to customers. This rise in demand leads to inflation, which finally becomes a necessary evil for a growing economy.

 

Question 15

The lowering of Bank Rate by the Reserve Bank of India leads to:

(a) More liquidity in the market

(b) Less liquidity in the market

(c) No change in the liquidity in the market

(d) Mobilization of more deposits by commercial banks

Ans: a         

Sub-Theme: Money and Banking/Monetary Policy

Option (a) is correct: Bank rate refers to the interest rate charged by the central bank (RBI) on loans granted to commercial banks. When Bank Rate is lowered by RBI, bank’s borrowing costs decrease (less interest) which in return, increases the supply of money in the market. It means more loans will be provided by banks, which will increase Liquidity in the market. On the other hand, an increase in this rate means that RBI is following a tight monetary policy as an increase in rates will lead to decrease in money circulation leading to a decrease in inflation.

 

Question 16

Which one of the following statements appropriately describes the “fiscal stimulus”?

(a) It is a massive investment by the Government in manufacturing sector to ensure the supply of goods to meet the demand surge caused by rapid economic growth

(b) It is an intense affirmative action of the Government to boost economic activity in the country

(c) It is Government’s intensive action on financial institutions to ensure disbursement of loans to agriculture and allied sectors to promote greater food production and contain food inflation

(d) It is an extreme affirmative action by the Government to pursue its policy of financial inclusion

Ans: b

Sub-Theme: Inflation/Monetary Policy/Fiscal

Policy/Money and Banking

Option (b) is correct: A ‘stimulus’ is an attempt by policymakers of a country to kickstart a sluggish economy through a package of measures. The central bank will use a monetary stimulus to boost consumer spending by increasing the money supply or lowering interest rates. A fiscal stimulus is when the government increases spending out of its own funds or lowers tax rates. Consumer spending increases as a result of stimulus measures, which boosts demand and growth. It’s common to refer to a stimulus as “priming the pump” or “pump priming.”

 

Question 17

A rapid increase in the rate of inflation is sometimes attributed to the “base effect”. What is “base effect”?

(a) It is the impact of drastic deficiency in supply due to failure of crops

(b) It is the impact of the surge in demand due to rapid economic growth

(c) It is the impact of the price levels of previous year on the calculation of inflation rate

(d) None of the statements (a), (b) and (c) given above is correct in this context

Ans: c

Sub-Theme: Inflation/Monetary Policy/Fiscal

Policy/Money and Banking

Option (c) is correct: Base Effect refers to the impact of an increase in the price level (i.e., previous year’s inflation) over the corresponding rise in price levels in the current year (i.e., current inflation). If the inflation rate was low in the corresponding period of the last year, then even a small increase in the price index will give a high rate of inflation in the current year.

 

Question 18

India is regarded as a country with a “Demographic Dividend”. This is due to:

(a) Its high population in the age group below 15 years

(b) Its high population in the group of 15-64 years

(c) Its high population in the age group above 65 years

(d) Its high total population

Ans: b

Sub-Theme: Demographic Dividend

Demographic Dividend:

According to United Nations Population Fund (UNFPA), demographic dividend means, “the economic growth potential that can result from shifts in a population’s age structure, mainly when the share of the working-age population (15 to 64) is larger than the non-working-age share of the population (14 and younger, and 65 and older)”.

 

Question 19

A “closed economy” is an economy in which:

(a) The money supply is fully controlled

(b) Deficit financing takes place

(c) Only exports take place

(d) Neither exports or imports take place

Ans: d          

Sub-Theme: Types of Economy

Refer to table below:

 

 

 

Closed Economy

•   Does not have economic relations with the rest of the world.

•   Activities taking place outside the territory do not affect the economic activities.

•   There is no difference b/w national income and domestic income.

•   It is an imaginary economy.

•   A closed economy is completely self-sufficient, which means that no imports enter and no exports leave the country.

•   E.g.: Brazil imports the least amount of goods in the world when measured as a portion of the gross domestic product (GDP) and is the most closed economy in the world.

 

 

Open Economy

•   An open economy has economic relations with other countries.

•   Economic activities of such an economy are affected by international fluctuations.

•   The size of national income may be greater or smaller than the domestic income.

•   It is a realistic economy.

•   E.g.: USA, Singapore, Finland, etc.

 

 Final Result – CIVIL SERVICES EXAMINATION, 2023.   Udaan-Prelims Wallah ( Static ) booklets 2024 released both in english and hindi : Download from Here!     Download UPSC Mains 2023 Question Papers PDF  Free Initiative links -1) Download Prahaar 3.0 for Mains Current Affairs PDF both in English and Hindi 2) Daily Main Answer Writing  , 3) Daily Current Affairs , Editorial Analysis and quiz ,  4) PDF Downloads  UPSC Prelims 2023 Trend Analysis cut-off and answer key

THE MOST
LEARNING PLATFORM

Learn From India's Best Faculty

      

 Final Result – CIVIL SERVICES EXAMINATION, 2023.   Udaan-Prelims Wallah ( Static ) booklets 2024 released both in english and hindi : Download from Here!     Download UPSC Mains 2023 Question Papers PDF  Free Initiative links -1) Download Prahaar 3.0 for Mains Current Affairs PDF both in English and Hindi 2) Daily Main Answer Writing  , 3) Daily Current Affairs , Editorial Analysis and quiz ,  4) PDF Downloads  UPSC Prelims 2023 Trend Analysis cut-off and answer key

Quick Revise Now !
AVAILABLE FOR DOWNLOAD SOON
UDAAN PRELIMS WALLAH
Comprehensive coverage with a concise format
Integration of PYQ within the booklet
Designed as per recent trends of Prelims questions
हिंदी में भी उपलब्ध
Quick Revise Now !
UDAAN PRELIMS WALLAH
Comprehensive coverage with a concise format
Integration of PYQ within the booklet
Designed as per recent trends of Prelims questions
हिंदी में भी उपलब्ध

<div class="new-fform">







    </div>

    Subscribe our Newsletter
    Sign up now for our exclusive newsletter and be the first to know about our latest Initiatives, Quality Content, and much more.
    *Promise! We won't spam you.
    Yes! I want to Subscribe.